SlideShare ist ein Scribd-Unternehmen logo
1 von 21
1. The increased intraocular tension              6. A 32-year-old patient has been diagnosed
is observed in a patient with glaucoma.           with bartholinitis (inflammation
Secretion of aqueous humor by the ciliar          of Bartholin's glands ). In what part of the
body is normal. Injury of what structure of       female urogenital system are the Bartholin's
the eyeball caused the disorder of flow-out       glands located?
from 'the anterior chamber?                       A. The vagina
A. Back epithelium of cornea                      B. The uterus
B. Choroid                                        C. The labia minor
C. Ciliary muscle                                 D. The clitoris
D. Venous sinus                                   E. The labia major
E. Ciliar body                                    7. The calcium canals of cardiomyocytes
2. A patient visited a dentist with complaints    have been blocked on an isolated rabbit's
of redness and edema of his mouth                 heart. What changes in the heart's activity
mucous membrane in a month after dental           can happen as a result?
prosthesis. The patient was diagnosed with        A. Decreased rate and force of heart beat
allergic stomatitis. What type of allergic        B. Heart stops in systole
reaction by Gell and Cumbs underlies this         C. Decreased force of the contraction
disease? .                                        D. Decreased heart beat rate
A. Cytotoxic                                      E. Heart stops in diastole
B. Immunocomplex                                  8. Substitution of the glutamic acid on
C. Anaphylactic                                   valine was revealed while examining initial
D. Delayed type hypersensitivity                  molecular structure. For what inherited
E. Stimulating                                    pathology is this symptom typical?
3. An autopsy revealed large (1-2 cm)             A. Thalassemia
brownish-red, easy crumbling formations           B. Sickle-cell anemia
covering ulcerative defects on the external       C. Favism
surface of the aortic valve. What is the most     D. Minkowsky-Shauffard disease
likely diagnosis?                                 E. Hemoglobinosis
A. Diffusive endocarditis                         9. A patient has undergone an amputation
B. Polypus-ulcerative endocarditis                of lower extremity. Some time later painful
C. Fibroplastic endocarditis                      nodules appeared in a stump. Amputatious
D. Acute warty endocarditis                       neuromas were found out at the microscopic
E. Recurrent warty endocarditis                   examination. To what pathological
4. A patient with suspicion on epidemic           processes do those formations relate?
typhus was admitted to the hospital. Some         A. Metaplasia
arachnids and insects have been found in          B. Inflammation
his flat. Which of them may be a carrier of       C. Regeneration
the pathogen of epidemic typhus?                  D. Dystrophy
A. Bed-bugs                                       E. Hyperemia
B. Houseflies                                     10. A patient with injured muscles of
C. Lice                                           the lower extremities was admitted to
D. Spiders                                        the traumatological department. Due to
E. Cockroaches                                    what cells is reparative regeneration of the
5. A 27- year-old woman has used penicillin       muscle fibers and restoration of the muscle
containing eye drops. In a few minutes itching,   function possible?
skin burning. lips and eyelids edema.             A. Satellite-cells
whistling cough. decreasing BP appeared.          B. Fibroblasts
What antibodies can lead to this allergic         C. Myoepithelial cells
reaction?                                         D. Myoblasts
A. IgE and IgG                                    E. Myofibroblasts
B. IgM and IgD                                    11. A worker has decreased buffer capacity
Co IgA and IgM                                    of blood due to exhausting muscular work
D. IgG and IgD                                    The influx of what acid substance in the
E. IgM and IgG                                    blood can cause this symptom?
A.3-phosphoglycerate                             the bars form three zones. Medulla consists
B.l,3-bisphosphoglycerate                        of chromaffinocytes and venous sinusoids.
C. Pyruvate                                      Which organ has these morphological
D. α-ketoglutarate                               features?
E. Lactate                                       A. Thyroid
12. A 63-year-old woman developed                B. Lymph node
symptoms of rheumatoid arthritis. Their          C. Kidney
increase of which blood values indicators        D. Adrenal gland
could be the most significant in proving the     E. Thymus
diagnosis?                                       17. The study of the genealogy of a family
A. Additive glycosaminoglycans                   with hypertrichosis (helix excessive pilosis)
B. Acid phosphatase                              has demonstrated that this symptom
C. Lipoproteids                                  is manifested in all generations only in men
D. General cholesterol                           and is inherited by son from his father. What
E. R-glycosidase                                 is the type of hypertrichosis inheritance?
13. A 50-year-old man has felt vague             A. Y-linked chromosome
abdominal discomfort within past 4               B. Autosome-dominant
months. Physical examination revealed            C. Autosome-recessive
no lymphadenopathy, and no abdominal             D. X-linked dominant chromosome
masses or organomegaly at palpation.             E. X-linked recessive chromosome
Bowel sounds are heard. An abdominal CT          18. Succinate dehydrogenase catalyses the
scan shows a 20 cm retroperitoneal soft tissue   dehydrogenation of succinate. Malonic acid
mass obscuring the left psoas muscle.            HOOC-CH2-COOH is used to interrupt
A stool specimen tested for occult blood is      the action of this enzyme. Choose the inhibition
negative. Which of the following neoplasms       type:
is this man most likely to have?                 A. Non-competitive
A. Adenocarcinoma                                B. Limited proteolysis
B. Lymphoma                                      C. Allosteric
C. Lipoma                                        D. Competitive
D. Melanoma                                      E. Dephosphorylation
E. Hamartoma                                     19. A patient has a malignisation of thoracic
14. The penetration of the irritable cell        part of esophagus. What lymphatic nodes
membrane for potassium ions has been             are regional for this organ?
increased during an experiment. What             A. Nodi lymphatici mediastinales posteriores
changes of membrane electric status can          B. Nodi lymphatici paratrachealis
occur?                                           C. Nodi lymphatici prevertebralis
A. Action potential                              D. Nodi lymphatici pericardiales laterales
B. Depolarization                                E. Anulus lymphaticus cardiae
C. Hyperpolarization                             20. A patient with bronchial asthma had
D. No changes                                    been taking tablets which caused insomnia,
E. Local response                                headache, increased blood pressure. What
15. Examination of a person revealed that        medecine can cause such complications?
minute volume of heart is 3500 mL, systolic      A. Ephedrine
volume is 50 mL. What is the frequency of        B. Chromolin sodium
cardiac contraction?                             C. Euphyline
A. 50bpm                                         D. Adrenaline
B. 70bpm                                         E. Izadrine
C. 80bpm                                         21. Inhibition of alpha-motoneuron of the
D. 60bpm                                         extensor muscles was noticed after stimulation
E. 90bpm                                         of a-motoneuron of the flexor
16. A histological spacemen presents             muscles during the experiment on the spinal
parenchymal organ, which has cortex and          column. What type of inhibition can be
medulla. Cortex consists of epitheliocytes       caused by this process?
bars with blood capillaries between them;        A. Depolarizational
B. Reciprocal                                    C. In the collecting duck
C. Lateral                                       D. In distal tubule of nephron
D. Presynaptic                                   E. In proximal tubule of nephron
E. Recurrent                                     27. A patient suffering from trombophlebitis
22. During histological examination of the       of deep veins suddenly died. The autopsy
stomach it was found out that glands contained   has shown freely lying red friable masses
very small amount of pariental cells or          with dim crimped surface in the trunk and
they were totally absent. Mucose membrane        bifurcation of the pulmonary artery. What
of what part of the stomach was studied')        pathologic process was revealed by the
A. -                                             morbid anatomist?
B. Cardia                                        A. Tissue embolism
C. Body of stomach                               B. Fat embolism
D. Pyloric part                                  C. Tromboembolism
E. Fundus of stomach                             D. Thrombosis
23. A businessman came to India from             E. Embolism with foreign body
South America. On examination the physician      28. On autopsy of a still-born infant
found that the patient was suffering             abnormalities have been revealed: ventricles
from sleeping-sickness. What was the way         are not separated, a single arterial trunk
of invasion?                                     originates from the right part. For what class
A. After contact with a sick dogs                of vertebrates is such heart construction
B. Through dirty hands                           characteristic?
C. As a result of mosquito's bites               A. Fishes
D. With contaminated fruits and vegetables       B. Birds
E. As a result of bug's bites                    C. Amphibian
24. A patient has been taking a mixture          D. Reptiles
prescribed by neuropathologist for               E. Mammals
neurasthenia for two weeks. The patient          29. A man after 1,5 litre blood loss has
feels better but has developed coryza,           suddenly reduced diuresis. TIle increased
conjunctivitis, rash, inertia, decrease of       secretion of what hormone caused such diuresis
memory. She is diagnosed with bromizm.           alteration?
What should be prescribed to decrease the        A. Parathormone
symptoms?                                        B. Corticotropin
A. Glucose solution 5%                           C. Vasopressin
B. Polyglucin                                    D. Cortisol
C. Asparcam                                      E. Natriuretic
D. Natrium chloride                              30. Blood analysis of a patient showed signs
E. -                                             of HIV infection (human immunodeficiency
25. When the pH level of the stomach lumen       virus). Which cells does HIV-virus primarily
decreases to less than 3, the antrum of          affect?
the stomach releases peptide that acts in        A. Specialized nervous cells (neurons)
paracrine fashion to inhibit gastrin release.    B. Cells that contain receptor IgM (B-
This peptide is:                                 lymphocytes)
A. Gastrin-releasing peptide (GRP)               C. Cells that contain receptor T4 (T-helpers)
B. GIF                                           D. Proliferating cells (stem hematoplastic
C. Somatostatin                                  cells)
D. Vasoactive intestinal peptide (VIP)           E. Mast cells
E. Acetylcholine                                 31. Slime, blood and protozoa 30-200 microns
26. The low specific gravity of the secondary    of length have been revealed in a
urine (1002) was found out in the sick           man's feces. The body is covered with cilias
person. What is the most distant part of         and has correct oval form with a little bit
nephron where concentration of secondary         narrowed forward and wide round shaped
urine takes place?                               back end. On the forward end a mouth is visible.
A. In the nephron's glomerulus                   In cytoplasm there are two nucleuses
B. In ascending part of loop of Henle
and two short vacuoles. For whom are the         degrees due to gamma radiation. What type
described attributes typical?                    of mutation took place in the DNA chain'?
A. Balantidium                                   A. Replication
B. Trichomonas                                   B. Inversion
C. Intestinal amoeba                             C. Translocation
D. Dysenteric amoeba                             D. Doubling
E. Lamblia                                       E. Deletion
32. A doctor administered Allopurinol to a       38. Periodic renal colics attacks are
26-year-old young man with the symptoms          observed in a woman with primer!
of gout. What pharmacological action of          hyperparathyroidizm. Ultrasonic examination
Allopurinol ensures therapeutical effect?        revealed small stones in the kidneys.
A. By increasing uric acid excretion             What is the most plausible reason of the
B. By inhibiting leucocyte migration into the    stones's formation?
joint                                            A. Hyperphosphatemia
C. By inhibiting uric acid synthesis             B. Hyperuricemia
D. By general analgetic effect                   C. Hypercalcemia
E. By general anti-inflammatory effect           D. Hyperkalemia
33. A person has steady HR not exceeding         E. Hypercholesterinemia
40 bpm. What is the pacemaker of the heart       39. The gluconeogenesis is activated in the
rhythm in this person?                           liver after intensive physical trainings .What
A. Atrioventricular node                         substance is utilized in gluconeogenesis first
B. His' bundle                                   of all in this case:
C. Sinoatrial node                               A. Glutamate
D. Purkinye' libel'S                             B. Pyruvate
E. Branches of His' bundle                       C. Lactate
34. In the ovary specimen colored with           D. Glucose
hematoxylin-eosin, follicle is determined        E. Alanine
where cubic-shaped follicle epithelium cells     40. From the nasopharynx of a 5-year-
are placed in 1-2 layers, and scarlet covering   old child 3 microorganism was excreted
is seen around ovocyte. Name this follicle:      which is identical to Corynebacterium diphtheriae
A. Primordial                                    dose according to morphological
B. Secondary                                     and biochemical signs. Microorganism does
C. Mature .                                      not produce exotoxin. As a result of what
D. Atretic                                       process can this microorganism become
E. Primary                                       toxigenic?
35. A denaturation of proteins can be found      A. Phage conversion
in some substances. Specify the substance        B. Passing through the organism of the
that is used for the incomplete denaturation     sensative animals
of hemoglobin:                                   C. Chromosome mutation
A. Toluene                                       D. Growing with antitoxic serum
B. Sulfuric acid                                 E. Cultivation in the telluric environment
C. Nitric acid                                   41. Necrosis focus was observed in the area
D. Urea                                          of hyperemia. and skin edema in a few hours
E. Sodium hydroxide                              after burn. What mechanism strengthens
36. Ovarian tumour was diagnozed in a            destructive effects in the inftamn1ation
woman. Surgery should be perfomed. What          area?
ligament should be extracted by the surgeon      A. Primary alteration
to disconnect the ovary and the uterus?          B. Emigration of lymphocytes
A. Lateral umbilical ligament                    C. Diapedesis of erythrocytes
B. Suspensory ligament of ovary                  D. Proliferation of fibroblasts
C. The ovarial ligament                          E. Secondary alteration
D. Round ligament of uterus                      42. There is an inhibited coagulation in the
E. Broad ligament of uterus                      patients with bile ducts obstruction, bleeding
37. Part of the DNA chain turned about 180       due to the low level of absorbtion of a
vitamin. What vitamin is in deficiency?             B. Escherichia
A. Carotene                                         C. Shigell
B. E                                                D. Streptococcus
C. K                                                E. Staphylococcus
D. A                                                48. A 58-year-old female has undergone
E. D                                                surgery for necrotic bowel. Despite having
43. In case of enterobiasis acrihine - the          been treated with antibiotics, on
structural analogue of vitamin B2 - is              postoperative day 5, she develops symptoms
administered.                                       (fever, hypotension, tachycardia, declining
The synthesis disorder of which                     urine output, and confusion) consistent with
enzymes does this medicine cause in                 septic shock. What hemodynamic support
microorganisms?                                     would be helpful?
A. NAD-dependet dehydrogenases                      A. Fluid administration
B. Cytochromeoxidases                               B. Dobutamine infusion
C. Aminotransferases                                C. Fluids and Dobutamine infusion
D. Peptidases                                       D. Atropine administration
E. FAD-dependent dehydrogenases                     E. Antibiotic administration
44. A patient with hypochromic anemia               49. A man died 8 days after the beginning
has splitting and loss of hair, increased nail      of the disease. He was diagnosed with
brittling and taste alteration. What IS the         dysentery. At the autopsy it was found
mechanism of the symptoms development?              out 'a thickened wall of the sigma and
A. Decreased production of parathyrin               rectum. fibrinous membrane on the surface
B. Deficiency of vitamin A                          of mucous membrane. Histologically: there
C. Deficiency of iron-containing enzymes            is a deep necrosis of mucous membrane with
D. Decreased production of thyroid                  infiltration of necrotic masses with fibrin.
hormones                                            What kind of colitis does correspond to the
E. Deficiency of vitamin B12                        changes?
45. Blood sampling for bulk analysis is             A. Catarrhal
recommended to be performed on an                   B. Diphtheritic
empty stomack and in the morning. What              C. Ulcerative
changes in blood composition can occur if to        D. Chronic
perform blood sampling after food intake?           E. Gangrenous
A. Increased plasma proteins                        50. The electronic microphoto of kidney
B. Increased contents of leukocytes                 fragment has exposed afferent glomerular
C. Reduced contents of erythrocytes                 arteriole, which has giant cells under its
D. Increased contents of erythrocytes               endothelium, containing secretory granules.
E. Reduced contents of thrombocytes                 Name the type of these cells:
46. The high level of Lactate Dehydrogenase         A. Interstitial
(LDH) isozymes concentration showed the             B. Mesangial
increase of LDH-l and LDH-2 in a patient's          C. Smoothmuscular
blood plasma. Point out the most probable           D. Juxtavascular
diagnosis:                                          E. Juxtaglomerular
A. Myocardial infarction                            51. When a patient with traumatic impairment
B. Acute pancreatitis                               of the brain was examined, it was discovered
C. Skeletal muscle dystrophy                        that he had stopped to distinguish
D. Viral hepatitis                                  displacement of an object on the skin. What
E. Diabetes mellitus                                part of the brain was damaged?
47. Red colonies spread in the large quantity       A. Posterior central gurus
in the Endo culture medium were revealed            B. Frontal zone
on bacteriological stool examination of a 4month-   C. Frontal central gurus
old baby with the symptoms of acute                 D. Parietal zone of the cortex
bowel infection. What microorganism can it          E. Occipital zone of the cortex
be?                                                 52. A 30-year-old patient was hospitalized
A. Salmonella                                       due to bleeding of the facial artery. What
place on the face has to be pressed to stop    by what of the following cells is the most
bleeding?                                      typical for the process occuring here?
 A. The mandible's branch                      A. Monocytes
B. The nose's back                             B. Limphocytes
C. The molar bone                              C. Neutrophils
D. The mental process                          D. Eosinophils
E. The mandible's edge                         E. Basophils
53. A patient with complaints of 3-day-long    58. Marked increase of activity of MB-forms
fever, general weakness, loss of appetite      of CPK (creatinephosphokinase) and
came to visit the infectionist. The doctor     LDH-l was revealed by examination of the
suspected enteric fever. Which method of       patient's blood. What is the most probab1e
laboratory diagnosis is the best to confirm    pathology?
the diagnosis?                                 A. Rheumatism
A. Detachment of myeloculture                  B. Hepatitis
B. Detachment of urine culture                 C. Miocardial infarction
C. Detachment of feces culture                 D. Pancreatitis
D. Detachment of blood culture                 E. Cholecystitis
E. Detachment of pure culture                  59. A patient who came to the doctor
54. A 50-year-old patient with typhoid fever   because of his infertility was administered
was treated with Levomycetin, the next day     to make tests for toxoplasmosis and chronic
his condition became worse, temperature        gonorrhoea. Which reaction should be
rose to 39,6°. What caused the complication?   performed to reveal latent toxoplasmosis
A. Secondary infection addition                and chronic gonurrhoea of the patient?
B. Irresponsiveness of an agent to the         A. IFA .. Immunofluorescence assay
levomycetin                                    B. RIHA - Reverse indirect hemagglutination
C. The effect of endotoxin agent               assay
D. Allergic reaction                           C. (R)CFT- Reiter's complement fixation
E. Reinfection                                 test
55. A student is thoroughly summarising a      D. RDHA - Reverse direct hemagglutination
lecture. When his groupmates begin talking     assay
the quality of the summarising worsens         E, Immunoblot analysis
greatly. What type of inhibition in the        60. A 45-year-old woman suffers from
cerebral cortex is the cause of it?            allergic seasonal COr'/Z2 caused by the
A. Differential                                ambrosia blossoming. What adipose cells
B. Dying                                       group stabilizer medicine can be used for
C. Protective                                  prevention of this disease?
D. External                                    A. Phencarol
E. Delayed                                     B. Diazoline
56. A 46 year-old man complains of difficult   C. Ketotifen
nose breathing. Mikulich cells, storage of     D. Dimedrol
epithelioid cells, plasmocytes, lymphocytes.   E. Tavegyl
hyaline balls are discovered in the biopsy     61. A patient after pathological process has
material of the nose thickening. What is the   a thickened alveolar membrane. The direct
most likely diagnosis?                         consequence of the process will be the
A. Virus rhinitis                              reduction of:
B. Rhinovirus infection                        A. Reserve expiratiory capacity
C. Meningococcal nasopharyngitis               B. Minute respiratory capacity
D. Allergic rhinitis                           C. Alveolar lung ventilation
E. Scleroma                                    Do Oxygen capacity of blood
57. A 16-year-old boy was performed an         E. Diffuse lung capacity
appendectomy. He has been hospitalized         62. A 59-year-old man has symptoms
for right lower quadrant abdominal pain        of parenchymatous jaundice and portal
within 18 hours. The surgical specimen is      hypertension. Histological examination
edematous and erythematous. Infiltration       of the puncture of the liver bioptate
has revealed an affected beam-lobule                 blood pressure. Increasing of the osmotic
structure, part of hepatocytes has signs of          concentration and dispersity of protein
fat dystrophy, port-portal connective tissue         structures can be found in the intercellular
septa with formation of pseudo-lobules, with         fluid. What kind of edema are to be
periportal lympho-macrophage infiltrations.          observed in this case?
What is the most probable diagnosis?                 A. Lymphogenic
A. Liver cirrhosis                                   B. Mixed
B. Alcohol hepatitis                                 C. Colloid-osmotic
C. Toxic dystrophy                                   D. Membranogenic
D. Chronic hepatosis                                 E. Hydrodynamic
E. Viral hepatitis                                   68. A patient died from acute cardiac
63. The preventive radioprotector was given          insufficiency. The histological examination
to a worker of a nuclear power station.              of his heart revealed the necrotized section
What mechanism from the below mentioned              in myocardium of the left ventricle, which
is considered to be the main mechanism               was separated from undamaged tissue
of radioprotection?                                  by the zone of hyperimic vessels, small
A. Activation of oxidation reactions                 hemorrhages and leukocytic infiltration.
B. Prevention of tissue's hypoxia                    What is the most likely diagnosis?
C. Increasing of tissue blood supply                 A. Myocardial infarction
D. Inhibition of free radicals formation             B. Myocardial ischemic dystrophy
E. Increasing of respiration                         C. Productive myocarditis
64. After the trauma. the patient's right            D. Focal exudate myocarditis
n.vagus was damaged. Which violation of              E. Diffuse exudate myocarditis
the cardiac activity is possible in this case?       69. A 56-year-old patient complaining of
A. Violation of a conductivity in the right          thirst and frequent urination was diagnosed
auricle                                              with diabete mellitus. Butamin was prescribed.
B. Violation of the automatism of a                  How does the medicine act?
atrioventricular                                     A. It inhibits α-cells of Langergans' islets
node                                                 B. It inhibits absorption of glucose in the
C. Block of a conductivity in the atrioventricular   intestines
node                                                 C. It relieves transport of glucose through
D. Violation of the automatism of a Kiss-Fleck       the cells' membranes
node                                                 D. It stimulates β-cells of Langergans' islets
E. Arrhythmia                                        E. It helps to absorb the glucose by the cells
65. A highly injured person has gradually died.      of the organism tissues
Please choose the indicator of biological            70. Galactosemia has been revealed in
death:                                               a child. Concentration of glucose in the
A. Disarray of chemical processes                    blood has not considerably changed. What
B. Autolysis and decay in the cells                  enzyme deficiency caused this illness?
C. Absence of palpitation and breathing              A. Galactokinase
D. Absence of movements                              B. Galactose-1-phosphate uridyltransferase
E. Loss of consciousness                             C. Hexokinase
66. During the experiment on the influence           D. Amylo-1,6-glucosidase
of chemical substances in the muscles the            E. Phosphoglucomutase
reaction of Ca2+ - pump is weakened. Which           71. During the endoscopy the inflammation
phenomenum will be observed?                         of a major papilla of the duodenum
A. Prolonged duration of the AP                      and the disturbances of bile secretion were
B. Decreased AP                                      found. In which part of duodenum were the
C. Prolonged relaxation                              problems found?
D. Decreased velocity of the AP distribution         A. Lower horizontal part
E. Activation of the sodium-potassium pump           B. Ascendant part
67. Inflamation is characterised by increasing       C. Descendent part
penetration of vessels of microcirculation           D. Upper horizontal part
stream, increasing of their fluid dynamic            E. Bulb
72. A 37-year-old man was admitted              and small-grained. Microscopically:
to the surgical department with the             fibrinogenous
symptoms of acute pancreatitis: vomiting,       inflammation of serous and
diarrhea, bradycardia, hypotention,             mucous capsules, dystrophic changes of
weakness, dehydration of the organism.          parenchymatous organs, brain edema. What
What medicine should be used first of all?      complication can cause such changes of
 A. Ephedrine                                   serous capsules and inner organs?
B. Platyphylline                                A. Sepsis
C. No-spa                                       B. Uraemia
D. Etaperazine                                  C. Thrombopenia
E. Contrycal                                    D. DIC-syndrome
73. Hydroxylation of endogenous substrates      E. Anemia
and xenobiotics requires a donor of protons.    78. A 13-year-old girl with history of
Which of the following vitamins can play        asthma complained of cough, dyspnea
this role?                                      and wheezing. Her symptoms became so
A. Vitamin B6                                   severe that her parents brought her to
B. Vitamin A                                    the emergency room. Physical examination
C. Vitamin P                                    revealed diaphoresis, dyspnea, tachycardia
D. Vitamin C                                    and tachypnea. Her respiratory rate was
E. Vitamin E                                    42/min. pulse rate was 110 beats per minute,
74. Obturative jaundice developed in a          and blood pressure was l30/70 mm
60-year-old patient because of malignant        Hg. Choose from the following list the
tumour of the big papillary of the duodenal.    most appropriate drug to reverse the
Lumen of what anatomical structure is           bronchoconstriction rapidly:
squeezed with tumour?                           A. Methylprednidsolone
A. Right hepatic duct                           B. Salbutamol
B. Common hepatic duct                          C. Beclomethasone
C. Hepatopancreatic ampulla                     D. Cromolyn
D. Cystic duct                                  E. Ipratropium
E. Left hepatic duct                            79. Chronic glomerulonephritis was diagnosed
75. A couple came for medical genetic           in a 34-year-old patient 3 years
counseling. The man has hemophilia, the         ago. Edema has developed within the last
woman is healthy and there were no cases        6 monthes. What caused the edema?
of hemophilia in her family. What is the risk   A. Hyperosmolarity of plasma
of having a sick child in this family?          B. Hyperaldosteronism
A.O                                             C. Liver disfunction of protein formation
B.25%                                           D. Hyperproduction of vasopressin
C.50%                                           E. Proteinuria
D.I00%                                          80. An isolated muscle of a frog is rhythmically
E. 75%                                          irritated with electric impulses. Every
76. Different functional groups can be          next impulse is in a period of relaxation
presented in the structure of L-amino acid's    from the previus contraction. What
radicals. Identify the group that is able       contraction of the muscle occurs?
to form ester bond:                             A. Continuous (smooth) tetanus
A. -CH3                                         B. Asynchronous
B.-CONH2                                        C. Single
C.-OH                                           D. Waved tetanus
D. -NH2                                         E. Tonic
E.-SH                                           81. The CNS stimulation produced by
77. For a long time a 49-year-old woman         methylxanthines, such as caffeine, is most
had suffered from glomerulonephritis which      likely due to the antagonism of one of the
caused death. The autopsy revealed              following recertors:
that the size of her kidneys was 7x3x2,5        A. Glycine receptors
sm, weight 65,0 g, they were dense              B. Adenosine receptors
C. Glutamate receptors                           injured?
D. GABA receptors                                A. The mind-brain
E. Cholinergic muscarinic receptors              B. The medulla oblongata
82. During complicated labour the                C. The cerebellum
symphysis pubis ruptured. What organ can         D. The inter-brain
be damaged mostly?                               E. The spinal cord
A. Uterus                                        87. The alveolar ventilation of the patient is
B. Rectum                                        5 L/min, the breath frequency is 10 per/min,
C. Ovaria                                        and the tidal volume is 700 ml. What is the
D. Urinary blader                                patient's dead space ventilation?
E. Uterine tubes                                 A. 2,0 L/min
83. A patient with abscess of the cut            B. 4,3 L/min
wound applied to the traumatological             c.-
department. The wound was washed with            D. 0,7 L/min
3% hydrogen peroxide to be cleaned from          E. 1,0 L/min
the pus. Foam was not observed. What             88. An old woman was hospitalized with
caused inefficiency of the drug?                 acute pain, edema in the right hip joint; the
A. Pus in the wound                              movements in the joint are limited. Which
B. Shallow wound                                 bone or part of it was broken?
C. Inherited insufficiency of catalase           A. Pubic bone
D. Low concentration H202                        B. The neck of the thigh
E. Inherited insufficiency erythrocyte's         C. Condyle of the thigh
phosphatdehydrogenase                            D. Ischial bone
84. A woman suffering from dysfunctional         E. The body of the thigh bone
metrorrhagia was made a diagnostic abortion.     89. A patient, who suffers from congenital
Histologically in the scrape there were          erythropoietic porphyria, has skin
a lot of small stamped glandulars covered        photosensitivity. The accumulation of what
with multirowed epithelium. The lumens of        compound in the skin can cause it?
some glandulars were cystically extended.        A. Heme
Choose the variant of general pathologic         B. Uroporphyrinogen 2
process in the endometrium.                      C. Coproporphyrinogen 3
A. Glandular-cystic hyperplasia of               D. Protoporphyrin
endometrium                                      E. Uroporphyrinogen 1
B. Neoplasm of endometrium                       90. A patient died 3 days after the operation
C. Atrophy of endometrium                        because of perforated colon with manifestations
D. Metaplasia of endometrium                     of diffuse purulent peritonitis.
E. Hypertrophic growth                           The autopsy revealed: colon mucos
85. An autopsy revealed: soft arachnoid          membrane was thickened and covered with
membrane of the upper parts of cerebral          a fibrin film, isolated ulcers penetrated at
hemisphere is plethoric, of yellowish-green      different
color, soaked with purulent and fibrose          depth. The histology result: mucous
exudate, it lookes like a cap. What disease is   membrane necrosis, leukocytes infiltration
characterised by these symtoms?                  with hemorrhages focuses. What disease
A. Tuberculous meningitis                        complication caused the patient's death?
B. Meningitis at typhus                          A. Dysentery
C. Influenza meningitis                          B. Crohn's disease
D. Meningitis at anthrax                         C. Amebiasis
E. Meningococcal meningitis                      D. Typhoid
86. A 50 year-old patien was injured on          E. Nonspecific ulcerative colitis
the occipital region of the head. The closed     91. The formation of a secondary mediator
skull's trauma was diagnosed. She was taken      is obligatory in membrane-intracellular
to the hospital. The medical examination:        mechanism of hormone action. Point out
deregulation of walking and balance.             the substance that is unable to be a
trembling of arms. What part of brain was        secondary mediator:
A. Glycerol                                           B. Anconeus muscle
B. Diacylglycerol                                     C. Tendon of the triceps muscle
C. CAMP                                               D. Aponeurosis of biceps muscle
D. CA2+                                               E. Brachioradial muscle '
E. Inositol-3,4,5-triphosphate                        97. The reason of occurrence of some diseases
92. A 45-year-old man with domestic apper             of an oral cavity is connected with
arm injuiry came to the trauma unit. The              structural peculiarities of its mucous
objective data are: there are no extension,           membrane. What morphological attributes
adduction or pronation functions of                   characterize these features?
the arm. What muscle damage caused this               A. Transitional epithelium, no submucosa
condition?                                            B. No muscularis mucosa, stratified
 A. Subscapular                                       squamous epithelium
B. Teres major                                        C. Transitional epithelium, no muscularis
C. Subspinous                                         mucosa
D. Supraspinous                                       D. Simple columnar ciliated epithelium
E. Teres minor                                        E. Well developed muscularis, no submucosa
93. The conjugated protein necessarily                98. A journalist's body temperature
contains special component as a nonprotein            has sharply increased in the morning
part. Choose the substance that                       three weeks after his mission in India,
can't carry out this function:                        it was accompanied with shivering and
A. Thiamine pyrophosphate                             bad headache. A few hours later the
B. Glucose                                            temperature decreased. The attacks began
C. HN03                                               to repeat in a day. He was diagnosed
D. AMP                                                with tropical malaria. What stage of
E. ATP                                                development of Plasmodium is infective for
94. M-r S presents all signs of the hepatic           anopheles-female?
coma: loss of consciousness, absence of               A. Merozoites
reflexes, cramps, convulsion, disorder of             B. Gametocytes
heart activity, recurrent (periodical) respiration.   C. Sporozoites
What are cerebrotoxical substances                    D. Microgamete
which accumulate in blood under hepar                 E. Shizontes
insufficiency?                                        99. A sick man with high temperature and
A. Autoantibody                                       a lot of tiny wounds on the body has been
B. IL-l                                               admitted to the hospital. Lice have been
C. Necrosogenic substances                            found in the folds of his clothing. What disease
D. Ammonia                                            can be suspected in the patient?
E. Ketonic body                                       A. Plague
95. A 60-year-old patient was hospitalised            B. Scabies
to the surgical department because                    c. Malaria
of infection caused by blue pus bacillus              D. Epidemic typhus
(Pseudomonas aeruginosa) which is sensative           E. Tularemia
to penicillin antibiotics. Indicate which of          100. The sterile Petri dishes and pipettes
the given penicillins has marked activity to          are necessary to prepare for microbiological
the Pseudomonas aeruginosa?                           tests in bacteriological laboratory. What
A. Oxacillin                                          way of sterilization should be applied in this
B. Benzylpenicillin                                   case?
C. Methicillin                                        A. Dry-heat sterilization
D. Phenoxymethylpenicillin                            B. Boiling
E. Carbenicillin disodium                             C. Pasteurization
96. Usually the intravenous injection is done         D. Tyndallization
into median cubital vein because it is slightly       E. Steam sterilizntion in autoclave
movable due to fixation by the soft                   101. An intraoperational biopsy of mammal
tissues. What does it fix in the cubital fossa?       gland has revealed the signs of atypical tissue
A. Brachial muscle                                    with disorder of parenchyma stroma
proportion with domination of the last,             E. Increased pressure in Bowman's capsule
gland structures of different size and shape,       106. If strong oxidizers get into the
lined with single-layer proliferative epithelium.   bloodstream. a methemoglobin is formed.
What is the most probable diagnosis?                It is a compound, where iron (II) becomes
A. Fibroadenoma                                     iron (III). What has to be done to save the
B. Papilloma                                        patient?
C. Noninfiltrative cancer                           A. He has to be calmed down and put to bed
D. Mastitis                                         B. Interchangeable hemotransfusion has to
E. Infiltrative cancer                              be done
102. A 10-year-old child complains of               C. He has to be given pure oxygen
weakness, nausea, irritability. Helminthes          D. Patient has to be exposed to the fresh air
of white color and 5-10 mm long have been           E. Respiratory centers have to be stimulated
found on the underwear. On microscopy of            107. Patient 54 year-old, 5th day
the scrape from the perianal folds achromic         after surgical operation. Blood count:
ova of the unsvmmetrical form have been             Erythrocytes 3, 6* 1012/l, Hemoglobin 95 g/l,
revealed. Which helminth is in the organism         Erythrocyte's hemoglobin content (color
of the child?                                       index) 0,78; Leukocytes 16 * 109/l, Platelets
A. Trichina                                         450*109/l. Blood picture: anizocytosis,
B. Enterobins vermicularis                          poikilocytosis,
C. Ascaris lumbricoides                             reticulocytes- 3,8%. What anemia
D. Ancylostoma duodenalis                           does this patient have?
E. Trichuris                                        A. Acquired hemolytic anemia
103. A 68-year-old woman can not move               B. Acute posthemorragic anemia
her upper and lower right extremities after         C. Hypoplastic anemia
stroke. Muscle tone of these extremities and        D. Anemia from iron deficiency
reflexes are increased. There are pathological      E. Chronic posthemorragic anemia
reflexes. What form of the paralysis is it?         108. A woman who was sick with rubella
A. Hemiplegia                                       during the pregnancy gave birth to a deaf
B. Dissociation                                     child with hare lip and cleft palate. This
C. Paraplegia                                       congenital defect is an example of:
D. Monoplegia                                       A. Down's syndrome
E. Tetraplegia                                      B. Edward's syndrome
104. After breathing with poisonous steams          C. Genocopy
there is an increased quantity of slime in          D. Phenocopy
respiratory passages of a chemical production       E. Patau's syndrome
worker. What of respiratory tract epithelial        109. A 22-year-old patient was admitted
cells participate in mucousa moistening?            to the hospital with complaints of heavy
A. Intercalated cells                               nasal breathing. During the examination
B. Goblet cells                                     of her nasal cavity the doctors found thickened
C. Endocrine cells                                  mucous membrane, a lot of mucus
D. Langergans cells                                 and nodular infiltrates without erosions in
E. Fibroblasts                                      the nose. The nasal rhinoscleroma was diagnosed.
105. A 16 year-old patient got numerous             The biopsy was taken. What typical
traumas in automobile accident. Now the             morphological changes may be found?
patient is having a shock. AP - 80/60 mm            A. Granulomas with Virchow's cells
Hg. daily urine volume 60-80 m!. What               B. Granulomas with foreign body cells
pathogenic mechanism leads to kidneys               C. Interstitial inflammation
function violation?                                 D. Granulomas with Mikulicz's cells
A. Decreased hydrostatic pressure m                 E. Granulomas with Langhan's cells
glomerular capillaries                              110. The alternate usage of dichlotiazide,
B. Increased vasopressin blood concentration        etacrin acid and lasex did not cause marked
C. Trauma of the urinary bladder                    diuretic effect in the patient with marked
D. Increased osmotic pressure in glomerular         peripheral edema. The aldosterone level in
capillaries                                         the blood is increased. Indicate which medicine
should be prescribed:                                E. Pathological anisocytosis
A. Amilorid                                          116. Glomerular filtration rate (GFR)
B. Urea                                              increased by 20% due to prolonged starvation
C. Clopamid                                          of the person. The most evident cause of
D. Mannit                                            filtration changes under this conditions is:
E. Spironolacton                                     A. Increase of filtration coefficient
111. A patient with tissue trauma was taken          B. Increase of penetration of the renal filter
a blood sample for the determination of              C. Decrease of oncotic pressure of blood
blood clotting parameters. Specify the right         plasma
sequence of extrinsic pathway activation.            D. Increase of systemic blood pressure
A. III- VIIa - Xa                                    E. Increase of renal plasma stream
B. IV - VIII: TF - Xa                                117. There is the change of teeth at the 6,8year-
C. IV - VIIa-Xa                                      old children: deciduous are replaced
D. III- VIII: TF - Xa                                by permanent. What embrionic tissues are
E. III-IV-Xa                                         the sources of formation of permanent teeth
112. A damage of the atomic power plant              tissues?
reactor resulted in the run-out of radioelements.    A. I, II brachial arches
People in the superstandard radiation zone were      B. Mesodermal epithelium and mesenhime
radiated with approximately                          C. Ectodermal epithelium of a tooth plate
250-300 r. and were immediately                      and Inesenhin1c
hospitalized. What changes in the blood              D. Entodermal epithelium of a tooth plate
count would be typical?                              and mesenhime
A. Lymphopenia                                       E. Entodermal epithelium and mesoderm
B. Neutropenia                                       118. A 50 year-old patient had hemorrhage
C. Leukopenia                                        of the brain and was taken to the hospital.
D. Thrombopenia                                      The place of hemorrhage was revealed
E. Anemia                                            on the lateral hemispheres surfaces during
113. Decreased blood supply to the organs            the medical examination. What artety was
causes hypoxia that activates fibroblasts            injured?
function. Volume of what elements is                 A. The middle cerebral artery
increased in this case?                              B. The posterior cerebral artery
A. Nerve elements                                    C. The anterior cerebral artery
B. Lymphatic vessels                                 D. The posterior communicating artery
C. Vessels of microcircular stream                   E. The anterior conmmunicating artery
D. Parenchymatous elements of the organ              119. While enrolling a child to school
E. Intercellular substance                           Mantu's test was made to define whether
114. Live vaccine is injected into the human         revaccination was needed. The test result is
body. Increasing activity of what cells of           negative. What does this test result mean?
connective tissue can be expected?                   A. Absence of antitoxic immunity to the
A. Fibroblasts and labrocytes                        tuberculosis
B. Adipocytes and adventitious cells                 B. Presence of cell immunity to the
C. Pigmentocytes and pericytes                       tuberculosis
D. Plasmocytes and lymphocytes                       C. Presence of antibodies for tubercle bacillus
E. Macrophages and fibroblasts                       D. Absence of antibodies for tubercle bacillus
115. In the blood of a 26-year-old man               E. Absence of cell immunity to the
18% of erythrocytes of the spherical, ball-shaped,   tuberculosis
flat and thorn-like shape have been                  120. Some diseases reveal symptoms
revealed. Other eritrocytes were in the form         of aldosteronism with hypertension and
of the concavo-concave disks. How is this            edema due to sodium retention in the
phenomenon called?                                   organism. What organ of the internal
A. Erytrocytosis                                     secretion is affected on aldosteronism?
B. Physiological anisocytosis                        A. Adrenal glands
C. Pathological poikilocytosis                       B. Pancreas
D. Physiological poikilocytosis                      C. Hypophysis
D. Testicle                                     A. Macrophages, monocytes
E. Ovaries                                      B. NK-cells
121. Oval and round organelles with double      C. Fibroblasts. T-Iymphocytes, B-lymphocytes
wall are seen at the electron micrograph.       D. T-Iymphocytes, B-Iymphocytes
The outer membrane is smooth, the inner         E. O-lymphocytes
membrane folded into cristae contain            126. An autopsy has revealed that kidneys
enzyme ATPase synthetase. These are:            are enlarged, surface is large-granular
A. Golgi complex                                because of multiple cavities with smooth
B. Centrioles                                   wall, which are filled with clear fluid. What
Co Mitochondria                                 kidney disease did the patient have?
D. Lysosomes                                    A. Pyelonephritis
E. Ribosomes                                    B. Infarction
122. A patient with encephalopathy was          C. Glomerulonephritis
admitted to neurological department.            D. Polycystic kidney
Correlation of increasing encephalopathy        E. Necrotic nephrosis
and substances absorbed by the                  127. During the operation on the hip joint
bloodstream from the intestines was revealed.   of a 5-year-old child her ligament was
What substances created in the                  damaged which caused bleeding. What ligament
intestines can cause endotoxemia?               was damaged?
A. Biotin                                       A. Iliofemoral
B. Indole                                       B. Ischiofemoral
C. Ornithine                                    C. The head of the thigh
D. Butyrate                                     D. Perpendicular of the acetabule
E. Acetacetate                                  E. Pubofemoral
123. A 19 year-old patient was diagnosed        128. A 57-year-old patient was admitted
with appendicitis and was hospitalized. The     to the gastroenterological department with
surgical operation on ablating appendix         suspicion of Zollinger-Ellison syndrom
vermiformis is to be performed. What            because of rapid increase of gastrin level in
artery must be fixed to stop bleeding during    the blood serum. What the most probable
the surgical operation?                         disorder of the secretory function of the
A. The colica media                             stomach here?
B. The iliac                                    A. Hypoacidity hyposecretion
C. The colica dextra                            B. Hyperacidity hypersecretion
D. The ileocolic artery                         C. Achylia
E. The colica sinistra                          D. Hypoacidity hypersecretion
124. Methotrexate (structural analogue of       E. Hyperacidity hyposecretion
the folic acid which is competitive inhibitor   129. A 2-year-old child has got intestinal
of the dihydrofolatreductase) is prescribed     dysbacteriosis, which results in hemorrhagic
for treatment of the malignant tumour.          syndrome. What is the most likely cause of
On which level does methotrexate hinder         hemorrhage of the child?
synthesis of the nucleic acids?                 A. Hypocalcemia
A. Processing                                   B. PP hypovitaminosis
B. Mononucleotide synthesis                     C. Activation of tissue thromboplastin
C. Transcription                                D. Fibrinogen deficiency
D. Reparation                                   E. Vitamin K insufficiency
E. Replication                                  130. A patient suffering from thyrotoxicosis
125. A patient with clinical signs              symptoms of vegetoasthenic syndrome
of immunodeficiency has unchanged               was revealed. What of the following would
number and functional activity of               show the histological appearance of a
T and B lymphocytes. Dysfunction's              thyroid gland being stimulated by
defect of antigen-presentation to the           thyroidstimulating
immunocompetent cells was found during          hormone (TSH)?
investigation on the molecule level. Defect     A. Columnar-shaped follicular cells
of what cells is the most probable here?        B. An abundance of colloid in the lumen of
the follicle                                        human blood sample is lower than normal.
C. Increased numbers of parafollicular cells        This leads to edema of tissues. What blood
D. Decreased numbers of parafollicular              function is damaged?
capillaries                                         A. Maintaining the Ph level
E. Decreased numbers of follicular cells            B. Maintaining the. blood sedimentation
131. The action of electric current on the          system
exitable cell caused depolarization of its          C. All answers are correct
membrane. Movement of what ions through             D. Maintaining the oncotic blood pressure
the membrane caused depolarisation?                 E. Maintaining the body temperature
A. Ca2+                                             136. In the microspecimen of red bone
B. HCO3-                                            marrow multiple capillares were revealed
C. K+                                               through the walls of which mature blood
D. CL-                                              cells penetrated. What type of capillares is
E. Na+                                              it?
132. A young man has an unpainfull                  A. Fenestrational
formation without marked borders in the             B. Somatical
soft tissues of his thigh. On the tissue bioptate   C. Visceral
the formation lookes like flesh of                  D. Sinusoidal
fish and consists of immature fibroblast-like       E. Lymphatic
cells with multiple mitosis growing through         137. A lung of a premature infant is
the muscles. What is the most likely diagnosis?     presented on electronic photomicrography
A. Myosarcoma                                       of biopsy material. Collapse of the alveolar
B. Cancer                                           wall caused by the deficiency of surfactant
c. Myoma                                            was revealed. Disfunction of what cells of
D. Fibrosarcoma                                     the alveolar wall caused it?
E. Fibroma                                          A. Secretory cells
133. A 46 year-old patient has complained           B. Alveolar macrophages
of headache, fatigue, thirst, pains in the spine    C. Alveocytes type II
and joints for the last 2 years. Clinically         D. Fibroblasts
observed disproportional enlargement of             E. Alveocytes type I
hands, feet, nose, superciliary arches. He          138. A patient had been taking glucocorticoids
notes that he needed to buy bigger shoes            for a long time. When the preparation
three times. What is the main reason of such        was withdrawn he developed the symptoms
disproportional enlargement of different            of disease aggravation, decreased blood
parts of the body?                                  pressure and weakness. What is the reason
A. Increased sensitivity of the tissues to          of this condition?
insulin .                                           A. Sensibilization
B. Joints dystrophy development                     B. Appearance of adrenal insufficiency
C. Cartilaginous tissue proliferation under         C. Cumulation
growth hormone influence                            n. Hyperproduction of ACTH
D. Increased sensitivity of the tissues to          E. Habituation
growth hormone                                      139. A 2-vear-old child experienced
E. Joints chronic inflammation development          convulsions because of lowering calcium
134. A tissue sample of benign tumor was            ions concentration in the blood plasma.
studied under the electron microscope. A            Function of what structure is decreased?
lot of small (15-20 nm) spherical bodies,           A. Parathyroid glands
consisting of 2 unequal subunits were               B. Hypophysis
detected. These are:                                C. Adrenal cortex
A. Microtubules                                     D. Pineal gland
B. Smooth endoplasmic reticulum                     E. Thymus
C. Ribosomes                                        140. During the fetal period of the
D. Mitochondria                                     development in the vascular system of
E. Golgi complex                                    the fetus a large arterial (Botallo's)
135. The concentration of albumins in               duct is functioning which converts into
lig .arteriosum after birth. What anatomical     B. Monocvtes
formations does this duct connect?               C. Eosinophiles
A. Aorta and superior vena cava                  D. Lymphocytes
B. Pulmonary trunk and aorta                     E. Platelets
C. Right and left auricle                        146. A 55-year-old patient with continuing
D. Aorta and inferior vena cava                  ventricular arrhythmias was admitted to
E. Pulmonary trunk and superior vena cava        the hospital. The patient is taking timolol
141. The B cells of endocrine portion of         drops for glaucoma, daily insulin injections
pancreas are selectively damaged by alloxan      for diabetes mellitus, and an ACE inhibitor
poisoning. How will it be reflected in blood     for hypertension. You have decided to use
plasma?                                          phenytoin instead of procainamide. What is
A. The content of globulins decreases            the reason?
B. The level of sugar decreases                  A. The hypertensive effects of procainamide
C. The content of albumins decreases             would aggravate the hypertension
D. The content of sugar increases                B. The anticholinergic effect of procainamide
E. The content of fibrinogen decrease            would aggravate glaucoma
142. A consumptive patient has an open           C. The cholinergic effects of procainamide
pulmonary form of disease. Choose what           would aggravate the diabetes
sputum staining should be selected for finding   D. The local anesthetic effect of procainamide
out the tubercle (Koch's) bacillus?              would potentiate diabetes
A. Method of Burry-Gins                          E. The local anesthetic effect of procainamide
B. Method of Ziel-Neelsen                        would aggravate the hypertension
C. Method of Gram                                147. Patient with diabetes mellitus experienced
D. Method of Romanowsky-Giemsa                   loss of consciousness and convulsions
E. Method of Neisser .                           after an injection of insulin. What might be
143. During the breakout of acute respiratory    the result of biochemical blood analysis for
infection in order to diagnose                   concentration of sugar?
influenza the express-diagnosis, based on        A. 3,3 mmol/L
revealing of specific viral antigen in the       B. 8,0 mmol/L
examined material (nasopharyngial lavage),       C. 10,0 mmol/L
is carried out. Which reaction is used for       D. 1.5 mmol/L
this?                                            E. 5,5 mmol/L
A. Opsonization                                  148. Moving of the daughter chromatids to
B. Complement binding                            the poles of the cell is observed in the mitotically
C. Immunofluorescence                            dividing cell. On what stage of the
D. Agglutination                                 mitotic cycle is this cell?
E. Precipitation                                 A. Anaphase
144. A 62-year-old patient was admitted to       B. Interfase
the neurological department due to cerebral      C. Prophase
haemorrage. His condition is grave. There is     D. Telophase
evident progression of deep and frequent         E. Metaphase
breath that turnes into reduction to apnoea      149. A 25-year-old woman with red and
and the cycle repeates. What respiration         itchy eczematoid dermatitis visits your
type has developed in the patient?               office, She had a dental procedure one
A. Gasping respiration                           day earlier with administration of a local
B. Biot's respiration                            anesthetic. There were no other findings,
C. Cheyne-Stockes respiration                    although she indicated that she had a history
D. Apneustic respiration                         of allergic reactions. Which of the
E. Kussmaul respiration                          following drugs is most likely involved?
145. Punctata hemorrhage was found out in        A. Procaine
the patient after application of a tourniquet.   B. Etidocaine
With disfunction of what blood cells is it       C. Bupivacaine
connected?                                       D. Cocaine
A. Neutrophiles                                  E. Lidocaine
150. A mother of a newborn complains                 C. Autosomal dominant
of her baby's constant belching with undigested      D. X-linked recessive
milk. Which developmental anomaly                    E. Y-linked
is it an evidence of?                                156. A patient's blood was analyzed and
A. Esophageal atresia                                the decreased erythrocyte's sedimentation
B. Esophageal fistula                                rate (ESR) was discovered. What disease
C. Labium leporium                                   from the listed below is accompanied with
D. Faux lupinum                                      decreased ESR?
E. Anal atresia                                      A. Vitamin B deficiency
151. Autopsy of the 58-year-old man hads             B. Hepatitis
revealed that mitral valve is deformed, thickened,   C. Polycytemia
does not totally close, Microscopically:             D. Myocardial infarction
centers of collagen fibers are eosinophilic,         E. Splenomegaly
have positive fibrin reaction. The                   157. The pulmonalis embolism has suddenly
most probable diagnosis is:                          developed in a 40 year-old patient with
A. Mucoid swelling                                   opened fracture of the hip. Choose the
B. Fibrinoid inflammation                            possible kind of embolism.
C. Amyloidosis                                       A. Thrombus-embolus
D. Fibrinoid swelling                                B. Air
E. Hyalinosis                                        C. Tissue
152. During surgical operation a blood               D. Foreign body
transfusion was made. The blood must be              E. Fat
checked to find antigens of some disease.            158. Where should the cathetor for
What disease is expected to be found?                evacuation of the lymph from the thoracic
A. Virus of hepatitis A                              lymph duct be inserted?
B. Enterovirus                                       A. To the inferior vena cava
C. Virus of hepatitis B                              B. To the right venous corner
D. Virus of hepatitis E                              C. To the left inguinal vein
E. Adenovirus                                        D. To the left venous corner
153. Electrocardiogram of a 45-year-old              E. To the superior vena cava
man showed absence of P-wave in all the              159. A 42-year-old man who has been
leads. What part of the conducting system is         injured in a car accident is brought into the
blocked?                                             emergency room. His blood alcohol level on
A. Atrioventricnlar node                             admission is 250 mg/dL. Hospital records
B. Common branch of the bundle of His                show a prior hospitalization for alcohol
C. Purkinje's fibres                                 related seizures. His wife confirms that he
D. Branches of the bundle of His                     has been drinking heavily for 3 weeks. What
E. Sinu-atrial node                                  treatment should be provided to the patient
154. Nowadays about 50 minor bases have              if he goes into withdrawal?
been found ill the t-RNA structure besides           A. None
the main four nitrogenous bases. Choose              B. Pentobarbital
the minor nitrogenous base:                          C. Phenobarbital
A. Adenine                                           D. Diazepam
B. Cytosine                                          E. Phenytoin
C. Cysteine                                          160. Synthesis of phospholipids is disordered
D. Uracil                                            under the liver fat infiltration.
E. Dihydrouracil                                     Indicate which of the following substances
155. A healthy woman has three sons                  can enhance the process of methylation
affected by color blindness who were born            during phospholipids synthesis?
after her two marriages. Children both of            A. Citrate
her husbands are healthy. What is the most           B. Ascorbic acid
possible pattern of inheritance of this disease?     C. Methionine
A. Autosomal recessive                               D. Glycerin
B. X-linked dominant                                 E. Glucose
161. A patient with the symptoms of acute           E. Mineralocorticoids
alcoholic poisoning was brought to the              165. A 60-year-old patient was diagnosed
hospital. What carbohydrates metabolism             with hypothalamic lateral nuclei stroke.
changes are typical for this condition?             What changes in patient's behavior may be
A. The g]uconeogenesis is increased in liver        expected?
B. The anaerobic glucose metabolism                 A. Depression
predominates in muscles                             B. Aggressive behaviour
C. The breakage of glycogen is increased in         C. Thirst
liver                                               D. Unsatisfied hunger
D. The anaerobic breakage of glucose is             E. The rejection of food
increased in muscles                                166. A 55-year-old patient was hospitalized
E. The gluconeogenesis velocity in liver is         in result of the trauma of the medial
decreased                                           group of femoral muscles. What kind of
162. An individual is characterized by              movements is the patient unable to do?
rounded face, broad forehead, a mongollan           A. Abduction of femur
type of eyelid fold, flattened nasal bridge,        B. Adduction of femur
permanently open mouth. projecting                  C. Flexion of femur
lower lip, protruding tongue, short neck, flat      D. Extension of femur
hands, and stubby fingers. What diagnosis           E. Suppination of femur
can be put to the patient?                          167. The process of heart transplantation
A. Klinefelter's syndrome                           determined the viability of myocardial cells.
B. Supermales                                       The determination of what myocardium
C. Down's syndrome                                  parameter is the most important'?
D. Alkaptonuria                                     A. Concentration of Ca-ions in heart vessels
E. Turner's syndrome                                B. Concentration of oxygen in heart vessels
163. A 40-year-old woman has had a feeling of       C. Rest potential of cardiomyocytes
abdominal discomfort for the past                   D. Concentration of calcium-ions in myofibrils
8 months. On pelvic examination, there is           E. Heart temperature
the right adnexal mass. Abdominal CT scan           168. A woman who was infected with
demonstrates a 7 Cm cystic mass involving           toxoplasmosis during the pregnancy has a
the right ovary with small areas of                 child with multiple congenital defects. This
calcification. The uterus is normal in size.        is a result of:
The right fallopian tube and ovary have             A. Recombination
been removed surgically. Grossly, the mass          B. Biological mutogenesis
on sectioning is filled with abundant hair          C. Teratogenesis
and sebum. Microscopically, the mass has            D. Chemical mutogenesis
glandular spaces lined by columnar epithelium,      E. Cancerogenesis
squamous epithelium with hair follicles             169. The energy inputs of a healthy man
cartilage, and dense connective tissue. What        have been measured. In what position was
type of tumour is it?                               the patient if his energy inputs were less
A. Teratoma                                         than the main exchange?
B. Squamous cell carcinoma of ovary                 A. Nervous exertion
C. Metastase of cervical carcinoma                  B. Easy work
D. Melanoma                                         C. Sleep
E. Sarcoma of ovary                                 D. Rest
164. A 19-year-old female suffers from              E. Calmness
tachycardia in rest condition, weight loss,         170. A patient after hypertension stroke
excessive sweating, exophtalmos and irritability.   does not have voluntary movements in his
What hormone would you expect to                    right arm and leg with the increased muscle
find elevated in her scrum?                         tone in these extremites. What type of disfunction
A. Cortisol                                         of nervous system is it?
B. Insulin                                          A. Peripheral paresis
C. Thyroxine                                        B. Peripheral paralysis
D.ACTH                                              C. Central paresis
D. Central paralysis                                Malaria. What method of laboratory diagnostics
E. Reflex paresis                                   is necessary to use?
171. The patient has come to the hospital           A. Immunological tests
from the smelting workshop in the condition         B. Blood examination
of hyperthermia. What is the direct cause           C. Urine examination
of loss of consciousness at the heat stroke?        D. Stool examination
A. Decreased brain blood supply                     E. Examination of vaginal and urethral
B. Dilatation of peripheral vessels                 discharge
C. Decrease of heart output                         176. The patient with pneumonia was
D. Arterial pressure drop                           treated with antibiotics for a long period,
E. Increased water loss through sweating            After treatment patient complains of
172. Scraps of the mycelium of a fungus,            frequent and watery stool, abdomenal pain.
spores, air bubbles and fat drops were discovered   What is the reason of intestine function disorder?
on microscopy of the patient's                      A. Bacteria toxins influence
hair excluded from the infected areas, What         B. Intestinal disbacteriosis development
fungus disease is characterised by this             C. Antibiotics toxic influence on the GIT
microscopic                                         D. Autoimmune reaction development
picture?                                            E. Hereditary enzyme defect
A. Epidermophytosis                                 177. Analeptical remedy of reflective type
B. Favus                                            from the H-cholinomimetics group was given
C. Trichophytosis                                   to the patient for restoration of breathing
D. Microspory                                       after poisoning with carbon monoxide,
E. Sporotrichosis                                   What medicine was prescribed to the patient?
173. A person was selling "homemade pork            A. Adrenalin hydrochloride
"sausages on the market. State sanitary             B. Atropine sulphate
inspector suspected falcification of the            c. Mesaton
sausages, What serological immune reaction          D. Pentamin
can identifiy food substance?                       E. Lobeline hydrochloride
A. Complement- fixation test                        178. A patient complains of frequent and
B. Agglutination test                               difficult urination, Imperfection of what
C. Precipitation test                               formation can cause it?
D. Immunofluorescence test                          A. Prostate
E. Indirect hemagglutination test                   B. Testicle adnexa
174. A 45-year-old man fell on the right            C. Testicles
knee and felt the acute pain in the joint On        D. Sperm bubbles
examination: severe edema on the anterior           E. Bulb-uretic glands
surface of the knee joint. Crunching sounds         179. A 10-year-old child complains of
are heard, while moving the joint. Which            weakness, nausea, irritability. Helminthes of
bone is destroyed?                                  while color and 5-10 mm long were found on
A. Left epicondyle of the thigh                     the underwear. On microscopy of the scrape
B. Knee-cap                                         from the perianal folds achromic ovums of
C. Head of the thigh bone                           unsymmetrical form were revealed. Indicate
D. Neck of the thigh bone                           what helminth is parasiting on the child?
E. Right epicondyle of the thigh                    A. Trichina
175. A patient has been brought to the              B. Trichuris
hospital with the complaints of headache,           C. Enterobins vermicularis
pain in left hypochondrium. He has been             D. Ancylostoma duodenalis
ill for 1,5 weeks, the sudden illness began         E. Ascaris lumbricoides
with the increase of body temperature up            180. A microscopic examination of the
to 39,9°C. In 3 hours the temperature               enlarged neck gland of a 14-year-old girl
decreased and hydropoiesis began. The               revealed destruction of the tissue structure
attacks repeat rhythmically in 48 hours.            of the node, absence of the lymph follicles,
The patient had visited one an African              sclerotic and necrosis parts. Cell
Country. The doctors have suspected                 constitution of the node is polymorphous,
lymphocites, eosinophiles, atypical cells        C. Decreased lungs elasticity
of the large size with mu1tiple-Iobule           D. Inhibition of respiratory center
nuclei (Beresovsky-Shternberg cells) and         E. Excitation of respiratory center
onenucleus large size cells are observed.        186. There is only One hormone among the
What is the most likely diagnosis?               neurohormones which refers to the derivatives
A. Fungous mycosis                               of amino acids according to classification.
B. Acute lympholeucosis                          Point it out:
C. Berkitt's lymphoma                            A. Somatotropin
D. Chronic lympholcucosis                        B. Oxytocin
E. Lymphogranulomatous                           C. Vasopressin
181. Only one factor can influence the           D. Melatonin
charge of amino acid radicals in the active      E. Thyroliberin
centre of enzyme, Name this factor:              187. At the "laboratory experiment
A. The surplus of a product                      the eukocyte culture was mixed with
B. pH medium                                     staphylococci. Neutrophile leukocytes
C. Temperature                                   engulfed and digested bacterial cells. This
O. The presence of a competitive inhibitor       processes are termed:
E. Pressure                                      A. Phagocytosis
182. The donor who had not donated the           B. Facilitated diffusion
blood for a long time was examined with          C. Pinocytosis
IFA method, Anti-HBs antibodies were             D. Osmosis
revealed, What does positive result of IFA       E. Diffusion
in this case mean?                               188. A 50-year-old male farm worker has
A. Chronic hepatitis C                           been brought to the emergency room. He
B. Acute hepatitis C                             was found confused in the orchard and since
C. Acute hepatitis B                             then has remained unconscious. His heart
D. Previous hepatitis B                          rate is 45 and his blood pressure is 80/40 mm
E. Chronic hepatitis B                           Hg. He is sweating and salivating profusely.
183. A patient has elbow joint trauma with       Which of the following should be prescribed?
avulsion of medial epicondyle of humerus.        A. Pentamine
What nerve can be damaged in this trauma?        B. Norepinephrine
A. Radial                                        C. Physostigmine
B. Medial cutaneous nerve of forearm             D. Atropine
C. Cardiac cutaneous nerve                       E. Proserine
D. Musculocutaneous nerve                        189. A young man felt sharp pain in the back
E. Ulnar                                         during active tightening on the horizontal
184. A 35-year-old man under the treatment       bar. Objectively: pain while moving upper
for pulmonary tuberculosis has acute-onset       extremity, limited pronation and adduction
of right big toe pain, swelling, and low-grade   functions. Sprain of what muscle can be
fever. The gouty arthritis was diagnosed and     observed here?
high serum uric acid level was found. Which      A. M.subscapularis
of the following antituberculosis drugs are      B. M.latissimus dorsi
known for causing high uric acid levels?         C. M.romboideus major
A. Rifampicin                                    D. M.levator scapulae
B. Aminosalicylic acid                           E. M.trapezius
C. Thiacetazone                                  190. Intrapleural pressure is being measured
D. Pyrazinamide                                  in a person. In what phase does a person
E. Cycloserine                                   hold his breath if the pressure is - 25 cm
185. X-rav examination discovered lungs          H20?
emphysema in the patient. What is the            A. Forced expiration
reason of short breath development in this       B. Quiet inspiration
case?                                            c.-
A. Increased lungs elasticity                    D. Forced inspiration
B. Decreasing of alveoli receptors sensitivity   E. Quiet expiration
191. To anaesthetize the manipulation             between tibia and fibula . What kind of bone
related to burn surface treatment, a patient      connection was injured?
was intravenously injected a medication           A. Fontanel
for short-acting narcosis. 1 minute               B. Ligament
later the patient being under anaesthesia         C. Membrane
had increased blood pressure, tachycardia,        D. Suture
increased tone of sceletal muscles; reflexes      E. Gomphosis
remained. After awakening the patient had         196. On autopsy a 35-year-old man the focus
desorientation and visual hallucinations.         of carnification 5 cm in diametre enclosed
What medication was the patient injected?         in a thin capsule was revealed in the second
A. Nitrous oxide                                  segment of the right lung. The focus consists
B. Thiopental sodium                              of a tough dry friable tissue with a
C. Ketamine                                       dim surface. For what disease are these
D. Diethyl ether                                  morphological changes typical?
E. Sombrevin                                      A. Lung cancer
192. The patient with diabetes mellitus has       B. Tumorous form of silicosis
been delivered in hospital in the state of        C. Postinftammatory pneumosclerosis
unconsciousness. Arterial pressure is low.        D. Tuberculoma
The patient has acidosis. Point substances,       E. Chondroma
which accumulation in the blood results in        197. A I8-year-old patient came to the outpatient
these manifestations:                             department wiith the complaints of
A. Cholesterol esters                             bleeding trauma in the vestibule of his nose.
B. High fatty acids                               On examination: the mechanical injure of
C. Monosaccharides                                the mucous laver of the vestibule without
D. Amino acids                                    continuation into nasal cavity proper. What
E. Ketone bodies                                  is the boundary between the vestibule and
193. A patient with hypersecretion of the         nasal cavity proper?
gastric juices was recomended to exclude          A. Nasal limen
concentrated bouillons and vegetable              B. Nostrils
decoctions from the diet because of their         C. Nasal septa
stimulation of gastric secretion. What is         D. Choanes
dominating                                        E. Nasal roller
mechanism of stimulation of secretion             198. A 52 year-old patient with bronchial
in this case?                                     asthma was treated with glucocorticoids.
A. Stimulation of excretion of secretion in the   Fever reaction appeared as a result of
duodenum                                          postinjective
B. Irritation of taste receptors                  abscess. The patient had subfebrile
C. Stimulation of gastrin production by           temperature, which didn't correspond
G-cells                                           to latitude and severity of inflamnmatory
D. Irritation of mechanoreceptors of the          process. Why did patient have low fever
stomach                                           reaction?
E. Irritation of mechanoreceptors of the oral     A. Inhibited endogen pyrogens production
cavity                                            B. Violation of heat-producing mechanisms
194. Pyruvate concentration in the patient's      C. Thermoregulation center inhibition
urine has increased 10 times from normal          D. Inflammatory barrier formation in injection
amount. What vitamin deficiency can be the        place
reason of this change:                            E. Violation of heat loss through lungs
A. Vitamin Bl                                     199. Purulent endometritis developed in a woman
B. Vitamin Bb                                     after delivery. Treating with antibiotics
C. Vitamin C                                      inhibitors of murein synthesis was
D. Vitamin A                                      ineffective. Wide spectrum bactericidal
E. Vitamin E                                      antibiotic was administered to her. In 6
195. A 6-year-old child fell on the cutting       hours temperature rapidly increased up
object and traumatized soft tissues               to 40°C with shiver. Muscle pains have
appeared. BP dropped down to 70/40
mmHg. Oligura has developed. What is the
main reason for the development of this
condition?
A. Toxic effect of preparation
B. Endotoxic shock
C. Internal bleedin
D. Anaphylactic shock
E. Bacteremia
200. A patient in three weeks after acute
myocardial infarction has pain in the heart
and joints and pneumonia. What is the
main mechanism of development of
postinfarction
Dressler's syndrome?
A. Autoimmune inflammation
B. Resorption of enzymes from necrotized
area of myocardium
C. Secondary infection
D. Ischemia of myocardium
E. Thrombosis vessels

Weitere ähnliche Inhalte

Was ist angesagt?

WISSEN MEDIQUIZ 2018 MAINS
WISSEN MEDIQUIZ 2018 MAINSWISSEN MEDIQUIZ 2018 MAINS
WISSEN MEDIQUIZ 2018 MAINSSAABAIIMSBBSR
 
Biology Test Review 1
Biology Test Review 1Biology Test Review 1
Biology Test Review 1Jeff Stokes
 
Biology Test Review 3
Biology Test Review 3Biology Test Review 3
Biology Test Review 3Jeff Stokes
 
Biology Test Review 2
Biology Test Review 2Biology Test Review 2
Biology Test Review 2Jeff Stokes
 
Krok 1 - 2014 Question Paper (Pharmacy)
Krok 1 - 2014 Question Paper (Pharmacy)Krok 1 - 2014 Question Paper (Pharmacy)
Krok 1 - 2014 Question Paper (Pharmacy)Eneutron
 

Was ist angesagt? (9)

AFMC Biology 2001
AFMC Biology  2001AFMC Biology  2001
AFMC Biology 2001
 
Dr. Terminology
Dr. TerminologyDr. Terminology
Dr. Terminology
 
Clinical cases april 7,13
Clinical cases april 7,13Clinical cases april 7,13
Clinical cases april 7,13
 
AFMC Biology 2000
AFMC Biology  2000AFMC Biology  2000
AFMC Biology 2000
 
WISSEN MEDIQUIZ 2018 MAINS
WISSEN MEDIQUIZ 2018 MAINSWISSEN MEDIQUIZ 2018 MAINS
WISSEN MEDIQUIZ 2018 MAINS
 
Biology Test Review 1
Biology Test Review 1Biology Test Review 1
Biology Test Review 1
 
Biology Test Review 3
Biology Test Review 3Biology Test Review 3
Biology Test Review 3
 
Biology Test Review 2
Biology Test Review 2Biology Test Review 2
Biology Test Review 2
 
Krok 1 - 2014 Question Paper (Pharmacy)
Krok 1 - 2014 Question Paper (Pharmacy)Krok 1 - 2014 Question Paper (Pharmacy)
Krok 1 - 2014 Question Paper (Pharmacy)
 

Andere mochten auch

Andere mochten auch (8)

Ophthalmo glossary
Ophthalmo glossaryOphthalmo glossary
Ophthalmo glossary
 
Evidence based medicine pdf
Evidence based medicine pdfEvidence based medicine pdf
Evidence based medicine pdf
 
162 classification of cardiac arrhythmias and conduction disturbances
162 classification of cardiac arrhythmias and conduction disturbances162 classification of cardiac arrhythmias and conduction disturbances
162 classification of cardiac arrhythmias and conduction disturbances
 
24 peptic ulcer
24 peptic ulcer24 peptic ulcer
24 peptic ulcer
 
Anaesthesia
AnaesthesiaAnaesthesia
Anaesthesia
 
Status asthmaticus
Status asthmaticusStatus asthmaticus
Status asthmaticus
 
Hookworm infection
Hookworm infectionHookworm infection
Hookworm infection
 
Gastric function tests
Gastric function testsGastric function tests
Gastric function tests
 

Ähnlich wie 2005

Krok 1 - 2007 Question Paper (General medicine)
Krok 1 - 2007 Question Paper (General medicine)Krok 1 - 2007 Question Paper (General medicine)
Krok 1 - 2007 Question Paper (General medicine)Eneutron
 
Krok 1 - 2015 (Histology)
Krok 1 - 2015 (Histology)Krok 1 - 2015 (Histology)
Krok 1 - 2015 (Histology)Eneutron
 
Krok 1 - 2012 Question Paper (Stomatology)
Krok 1 - 2012 Question Paper (Stomatology)Krok 1 - 2012 Question Paper (Stomatology)
Krok 1 - 2012 Question Paper (Stomatology)Eneutron
 
Krok 1 - 2015 Question Paper (Stomatology)
Krok 1 - 2015 Question Paper (Stomatology)Krok 1 - 2015 Question Paper (Stomatology)
Krok 1 - 2015 Question Paper (Stomatology)Eneutron
 
Krok 1 - 2008 Question Paper (General medicine)
Krok 1 - 2008 Question Paper (General medicine)Krok 1 - 2008 Question Paper (General medicine)
Krok 1 - 2008 Question Paper (General medicine)Eneutron
 
Krok 1 - 2012 Question Paper (General medicine)
Krok 1 - 2012 Question Paper (General medicine)Krok 1 - 2012 Question Paper (General medicine)
Krok 1 - 2012 Question Paper (General medicine)Eneutron
 
Krok 1 - 2015 Question Paper (General medicine)
Krok 1 - 2015 Question Paper (General medicine)Krok 1 - 2015 Question Paper (General medicine)
Krok 1 - 2015 Question Paper (General medicine)Eneutron
 
Krok 1 - 2014 (Path-Physiology)
Krok 1 - 2014 (Path-Physiology)Krok 1 - 2014 (Path-Physiology)
Krok 1 - 2014 (Path-Physiology)Eneutron
 
Krok 1 - 2011 Question Paper (General medicine)
Krok 1 - 2011 Question Paper (General medicine)Krok 1 - 2011 Question Paper (General medicine)
Krok 1 - 2011 Question Paper (General medicine)Eneutron
 
Krok 1 - 2013 Question Paper (Stomatology)
Krok 1 - 2013 Question Paper (Stomatology)Krok 1 - 2013 Question Paper (Stomatology)
Krok 1 - 2013 Question Paper (Stomatology)Eneutron
 
Krok 1 - 2014 (Histology)
Krok 1 - 2014 (Histology)Krok 1 - 2014 (Histology)
Krok 1 - 2014 (Histology)Eneutron
 
Krok1 stomatology - 2016
Krok1   stomatology - 2016Krok1   stomatology - 2016
Krok1 stomatology - 2016Eneutron
 
Krok 1 - 2014 Question Paper (General medicine)
Krok 1 - 2014 Question Paper (General medicine)Krok 1 - 2014 Question Paper (General medicine)
Krok 1 - 2014 Question Paper (General medicine)Eneutron
 
Krok 1 - 2007 Question Paper (Stomatology)
Krok 1 - 2007 Question Paper (Stomatology)Krok 1 - 2007 Question Paper (Stomatology)
Krok 1 - 2007 Question Paper (Stomatology)Eneutron
 
Krok 1 - 2015 (Path-Physiology)
Krok 1 - 2015 (Path-Physiology)Krok 1 - 2015 (Path-Physiology)
Krok 1 - 2015 (Path-Physiology)Eneutron
 
Krok 1 - 2015 (Anatomy)
Krok 1 - 2015 (Anatomy)Krok 1 - 2015 (Anatomy)
Krok 1 - 2015 (Anatomy)Eneutron
 
Krok 1 - 2005 Question Paper (General medicine)
Krok 1 - 2005 Question Paper (General medicine)Krok 1 - 2005 Question Paper (General medicine)
Krok 1 - 2005 Question Paper (General medicine)Eneutron
 
Krok 1 - 2006 Question Paper (Stomatology)
Krok 1 - 2006 Question Paper (Stomatology)Krok 1 - 2006 Question Paper (Stomatology)
Krok 1 - 2006 Question Paper (Stomatology)Eneutron
 

Ähnlich wie 2005 (20)

2007
20072007
2007
 
Krok 1 - 2007 Question Paper (General medicine)
Krok 1 - 2007 Question Paper (General medicine)Krok 1 - 2007 Question Paper (General medicine)
Krok 1 - 2007 Question Paper (General medicine)
 
file
file file
file
 
Krok 1 - 2015 (Histology)
Krok 1 - 2015 (Histology)Krok 1 - 2015 (Histology)
Krok 1 - 2015 (Histology)
 
Krok 1 - 2012 Question Paper (Stomatology)
Krok 1 - 2012 Question Paper (Stomatology)Krok 1 - 2012 Question Paper (Stomatology)
Krok 1 - 2012 Question Paper (Stomatology)
 
Krok 1 - 2015 Question Paper (Stomatology)
Krok 1 - 2015 Question Paper (Stomatology)Krok 1 - 2015 Question Paper (Stomatology)
Krok 1 - 2015 Question Paper (Stomatology)
 
Krok 1 - 2008 Question Paper (General medicine)
Krok 1 - 2008 Question Paper (General medicine)Krok 1 - 2008 Question Paper (General medicine)
Krok 1 - 2008 Question Paper (General medicine)
 
Krok 1 - 2012 Question Paper (General medicine)
Krok 1 - 2012 Question Paper (General medicine)Krok 1 - 2012 Question Paper (General medicine)
Krok 1 - 2012 Question Paper (General medicine)
 
Krok 1 - 2015 Question Paper (General medicine)
Krok 1 - 2015 Question Paper (General medicine)Krok 1 - 2015 Question Paper (General medicine)
Krok 1 - 2015 Question Paper (General medicine)
 
Krok 1 - 2014 (Path-Physiology)
Krok 1 - 2014 (Path-Physiology)Krok 1 - 2014 (Path-Physiology)
Krok 1 - 2014 (Path-Physiology)
 
Krok 1 - 2011 Question Paper (General medicine)
Krok 1 - 2011 Question Paper (General medicine)Krok 1 - 2011 Question Paper (General medicine)
Krok 1 - 2011 Question Paper (General medicine)
 
Krok 1 - 2013 Question Paper (Stomatology)
Krok 1 - 2013 Question Paper (Stomatology)Krok 1 - 2013 Question Paper (Stomatology)
Krok 1 - 2013 Question Paper (Stomatology)
 
Krok 1 - 2014 (Histology)
Krok 1 - 2014 (Histology)Krok 1 - 2014 (Histology)
Krok 1 - 2014 (Histology)
 
Krok1 stomatology - 2016
Krok1   stomatology - 2016Krok1   stomatology - 2016
Krok1 stomatology - 2016
 
Krok 1 - 2014 Question Paper (General medicine)
Krok 1 - 2014 Question Paper (General medicine)Krok 1 - 2014 Question Paper (General medicine)
Krok 1 - 2014 Question Paper (General medicine)
 
Krok 1 - 2007 Question Paper (Stomatology)
Krok 1 - 2007 Question Paper (Stomatology)Krok 1 - 2007 Question Paper (Stomatology)
Krok 1 - 2007 Question Paper (Stomatology)
 
Krok 1 - 2015 (Path-Physiology)
Krok 1 - 2015 (Path-Physiology)Krok 1 - 2015 (Path-Physiology)
Krok 1 - 2015 (Path-Physiology)
 
Krok 1 - 2015 (Anatomy)
Krok 1 - 2015 (Anatomy)Krok 1 - 2015 (Anatomy)
Krok 1 - 2015 (Anatomy)
 
Krok 1 - 2005 Question Paper (General medicine)
Krok 1 - 2005 Question Paper (General medicine)Krok 1 - 2005 Question Paper (General medicine)
Krok 1 - 2005 Question Paper (General medicine)
 
Krok 1 - 2006 Question Paper (Stomatology)
Krok 1 - 2006 Question Paper (Stomatology)Krok 1 - 2006 Question Paper (Stomatology)
Krok 1 - 2006 Question Paper (Stomatology)
 

Mehr von Bahaa A

2013krok
2013krok2013krok
2013krokBahaa A
 
pediatric сase history
pediatric сase historypediatric сase history
pediatric сase historyBahaa A
 
Sport medicine
Sport medicineSport medicine
Sport medicineBahaa A
 
Acute pancreatitis
Acute pancreatitisAcute pancreatitis
Acute pancreatitisBahaa A
 

Mehr von Bahaa A (8)

2013krok
2013krok2013krok
2013krok
 
pediatric сase history
pediatric сase historypediatric сase history
pediatric сase history
 
Sport medicine
Sport medicineSport medicine
Sport medicine
 
Acute pancreatitis
Acute pancreatitisAcute pancreatitis
Acute pancreatitis
 
Urology
UrologyUrology
Urology
 
Diary
Diary Diary
Diary
 
2010
20102010
2010
 
2009
20092009
2009
 

Kürzlich hochgeladen

Using Grammatical Signals Suitable to Patterns of Idea Development
Using Grammatical Signals Suitable to Patterns of Idea DevelopmentUsing Grammatical Signals Suitable to Patterns of Idea Development
Using Grammatical Signals Suitable to Patterns of Idea Developmentchesterberbo7
 
How to Make a Duplicate of Your Odoo 17 Database
How to Make a Duplicate of Your Odoo 17 DatabaseHow to Make a Duplicate of Your Odoo 17 Database
How to Make a Duplicate of Your Odoo 17 DatabaseCeline George
 
Mental Health Awareness - a toolkit for supporting young minds
Mental Health Awareness - a toolkit for supporting young mindsMental Health Awareness - a toolkit for supporting young minds
Mental Health Awareness - a toolkit for supporting young mindsPooky Knightsmith
 
Team Lead Succeed – Helping you and your team achieve high-performance teamwo...
Team Lead Succeed – Helping you and your team achieve high-performance teamwo...Team Lead Succeed – Helping you and your team achieve high-performance teamwo...
Team Lead Succeed – Helping you and your team achieve high-performance teamwo...Association for Project Management
 
Daily Lesson Plan in Mathematics Quarter 4
Daily Lesson Plan in Mathematics Quarter 4Daily Lesson Plan in Mathematics Quarter 4
Daily Lesson Plan in Mathematics Quarter 4JOYLYNSAMANIEGO
 
INTRODUCTION TO CATHOLIC CHRISTOLOGY.pptx
INTRODUCTION TO CATHOLIC CHRISTOLOGY.pptxINTRODUCTION TO CATHOLIC CHRISTOLOGY.pptx
INTRODUCTION TO CATHOLIC CHRISTOLOGY.pptxHumphrey A Beña
 
Oppenheimer Film Discussion for Philosophy and Film
Oppenheimer Film Discussion for Philosophy and FilmOppenheimer Film Discussion for Philosophy and Film
Oppenheimer Film Discussion for Philosophy and FilmStan Meyer
 
Student Profile Sample - We help schools to connect the data they have, with ...
Student Profile Sample - We help schools to connect the data they have, with ...Student Profile Sample - We help schools to connect the data they have, with ...
Student Profile Sample - We help schools to connect the data they have, with ...Seán Kennedy
 
ROLES IN A STAGE PRODUCTION in arts.pptx
ROLES IN A STAGE PRODUCTION in arts.pptxROLES IN A STAGE PRODUCTION in arts.pptx
ROLES IN A STAGE PRODUCTION in arts.pptxVanesaIglesias10
 
Expanded definition: technical and operational
Expanded definition: technical and operationalExpanded definition: technical and operational
Expanded definition: technical and operationalssuser3e220a
 
Transaction Management in Database Management System
Transaction Management in Database Management SystemTransaction Management in Database Management System
Transaction Management in Database Management SystemChristalin Nelson
 
Unraveling Hypertext_ Analyzing Postmodern Elements in Literature.pptx
Unraveling Hypertext_ Analyzing  Postmodern Elements in  Literature.pptxUnraveling Hypertext_ Analyzing  Postmodern Elements in  Literature.pptx
Unraveling Hypertext_ Analyzing Postmodern Elements in Literature.pptxDhatriParmar
 
Measures of Position DECILES for ungrouped data
Measures of Position DECILES for ungrouped dataMeasures of Position DECILES for ungrouped data
Measures of Position DECILES for ungrouped dataBabyAnnMotar
 
ClimART Action | eTwinning Project
ClimART Action    |    eTwinning ProjectClimART Action    |    eTwinning Project
ClimART Action | eTwinning Projectjordimapav
 
4.11.24 Mass Incarceration and the New Jim Crow.pptx
4.11.24 Mass Incarceration and the New Jim Crow.pptx4.11.24 Mass Incarceration and the New Jim Crow.pptx
4.11.24 Mass Incarceration and the New Jim Crow.pptxmary850239
 
MS4 level being good citizen -imperative- (1) (1).pdf
MS4 level   being good citizen -imperative- (1) (1).pdfMS4 level   being good citizen -imperative- (1) (1).pdf
MS4 level being good citizen -imperative- (1) (1).pdfMr Bounab Samir
 
BIOCHEMISTRY-CARBOHYDRATE METABOLISM CHAPTER 2.pptx
BIOCHEMISTRY-CARBOHYDRATE METABOLISM CHAPTER 2.pptxBIOCHEMISTRY-CARBOHYDRATE METABOLISM CHAPTER 2.pptx
BIOCHEMISTRY-CARBOHYDRATE METABOLISM CHAPTER 2.pptxSayali Powar
 
4.11.24 Poverty and Inequality in America.pptx
4.11.24 Poverty and Inequality in America.pptx4.11.24 Poverty and Inequality in America.pptx
4.11.24 Poverty and Inequality in America.pptxmary850239
 
Q-Factor General Quiz-7th April 2024, Quiz Club NITW
Q-Factor General Quiz-7th April 2024, Quiz Club NITWQ-Factor General Quiz-7th April 2024, Quiz Club NITW
Q-Factor General Quiz-7th April 2024, Quiz Club NITWQuiz Club NITW
 

Kürzlich hochgeladen (20)

Using Grammatical Signals Suitable to Patterns of Idea Development
Using Grammatical Signals Suitable to Patterns of Idea DevelopmentUsing Grammatical Signals Suitable to Patterns of Idea Development
Using Grammatical Signals Suitable to Patterns of Idea Development
 
How to Make a Duplicate of Your Odoo 17 Database
How to Make a Duplicate of Your Odoo 17 DatabaseHow to Make a Duplicate of Your Odoo 17 Database
How to Make a Duplicate of Your Odoo 17 Database
 
Mental Health Awareness - a toolkit for supporting young minds
Mental Health Awareness - a toolkit for supporting young mindsMental Health Awareness - a toolkit for supporting young minds
Mental Health Awareness - a toolkit for supporting young minds
 
Team Lead Succeed – Helping you and your team achieve high-performance teamwo...
Team Lead Succeed – Helping you and your team achieve high-performance teamwo...Team Lead Succeed – Helping you and your team achieve high-performance teamwo...
Team Lead Succeed – Helping you and your team achieve high-performance teamwo...
 
Daily Lesson Plan in Mathematics Quarter 4
Daily Lesson Plan in Mathematics Quarter 4Daily Lesson Plan in Mathematics Quarter 4
Daily Lesson Plan in Mathematics Quarter 4
 
INTRODUCTION TO CATHOLIC CHRISTOLOGY.pptx
INTRODUCTION TO CATHOLIC CHRISTOLOGY.pptxINTRODUCTION TO CATHOLIC CHRISTOLOGY.pptx
INTRODUCTION TO CATHOLIC CHRISTOLOGY.pptx
 
Oppenheimer Film Discussion for Philosophy and Film
Oppenheimer Film Discussion for Philosophy and FilmOppenheimer Film Discussion for Philosophy and Film
Oppenheimer Film Discussion for Philosophy and Film
 
INCLUSIVE EDUCATION PRACTICES FOR TEACHERS AND TRAINERS.pptx
INCLUSIVE EDUCATION PRACTICES FOR TEACHERS AND TRAINERS.pptxINCLUSIVE EDUCATION PRACTICES FOR TEACHERS AND TRAINERS.pptx
INCLUSIVE EDUCATION PRACTICES FOR TEACHERS AND TRAINERS.pptx
 
Student Profile Sample - We help schools to connect the data they have, with ...
Student Profile Sample - We help schools to connect the data they have, with ...Student Profile Sample - We help schools to connect the data they have, with ...
Student Profile Sample - We help schools to connect the data they have, with ...
 
ROLES IN A STAGE PRODUCTION in arts.pptx
ROLES IN A STAGE PRODUCTION in arts.pptxROLES IN A STAGE PRODUCTION in arts.pptx
ROLES IN A STAGE PRODUCTION in arts.pptx
 
Expanded definition: technical and operational
Expanded definition: technical and operationalExpanded definition: technical and operational
Expanded definition: technical and operational
 
Transaction Management in Database Management System
Transaction Management in Database Management SystemTransaction Management in Database Management System
Transaction Management in Database Management System
 
Unraveling Hypertext_ Analyzing Postmodern Elements in Literature.pptx
Unraveling Hypertext_ Analyzing  Postmodern Elements in  Literature.pptxUnraveling Hypertext_ Analyzing  Postmodern Elements in  Literature.pptx
Unraveling Hypertext_ Analyzing Postmodern Elements in Literature.pptx
 
Measures of Position DECILES for ungrouped data
Measures of Position DECILES for ungrouped dataMeasures of Position DECILES for ungrouped data
Measures of Position DECILES for ungrouped data
 
ClimART Action | eTwinning Project
ClimART Action    |    eTwinning ProjectClimART Action    |    eTwinning Project
ClimART Action | eTwinning Project
 
4.11.24 Mass Incarceration and the New Jim Crow.pptx
4.11.24 Mass Incarceration and the New Jim Crow.pptx4.11.24 Mass Incarceration and the New Jim Crow.pptx
4.11.24 Mass Incarceration and the New Jim Crow.pptx
 
MS4 level being good citizen -imperative- (1) (1).pdf
MS4 level   being good citizen -imperative- (1) (1).pdfMS4 level   being good citizen -imperative- (1) (1).pdf
MS4 level being good citizen -imperative- (1) (1).pdf
 
BIOCHEMISTRY-CARBOHYDRATE METABOLISM CHAPTER 2.pptx
BIOCHEMISTRY-CARBOHYDRATE METABOLISM CHAPTER 2.pptxBIOCHEMISTRY-CARBOHYDRATE METABOLISM CHAPTER 2.pptx
BIOCHEMISTRY-CARBOHYDRATE METABOLISM CHAPTER 2.pptx
 
4.11.24 Poverty and Inequality in America.pptx
4.11.24 Poverty and Inequality in America.pptx4.11.24 Poverty and Inequality in America.pptx
4.11.24 Poverty and Inequality in America.pptx
 
Q-Factor General Quiz-7th April 2024, Quiz Club NITW
Q-Factor General Quiz-7th April 2024, Quiz Club NITWQ-Factor General Quiz-7th April 2024, Quiz Club NITW
Q-Factor General Quiz-7th April 2024, Quiz Club NITW
 

2005

  • 1. 1. The increased intraocular tension 6. A 32-year-old patient has been diagnosed is observed in a patient with glaucoma. with bartholinitis (inflammation Secretion of aqueous humor by the ciliar of Bartholin's glands ). In what part of the body is normal. Injury of what structure of female urogenital system are the Bartholin's the eyeball caused the disorder of flow-out glands located? from 'the anterior chamber? A. The vagina A. Back epithelium of cornea B. The uterus B. Choroid C. The labia minor C. Ciliary muscle D. The clitoris D. Venous sinus E. The labia major E. Ciliar body 7. The calcium canals of cardiomyocytes 2. A patient visited a dentist with complaints have been blocked on an isolated rabbit's of redness and edema of his mouth heart. What changes in the heart's activity mucous membrane in a month after dental can happen as a result? prosthesis. The patient was diagnosed with A. Decreased rate and force of heart beat allergic stomatitis. What type of allergic B. Heart stops in systole reaction by Gell and Cumbs underlies this C. Decreased force of the contraction disease? . D. Decreased heart beat rate A. Cytotoxic E. Heart stops in diastole B. Immunocomplex 8. Substitution of the glutamic acid on C. Anaphylactic valine was revealed while examining initial D. Delayed type hypersensitivity molecular structure. For what inherited E. Stimulating pathology is this symptom typical? 3. An autopsy revealed large (1-2 cm) A. Thalassemia brownish-red, easy crumbling formations B. Sickle-cell anemia covering ulcerative defects on the external C. Favism surface of the aortic valve. What is the most D. Minkowsky-Shauffard disease likely diagnosis? E. Hemoglobinosis A. Diffusive endocarditis 9. A patient has undergone an amputation B. Polypus-ulcerative endocarditis of lower extremity. Some time later painful C. Fibroplastic endocarditis nodules appeared in a stump. Amputatious D. Acute warty endocarditis neuromas were found out at the microscopic E. Recurrent warty endocarditis examination. To what pathological 4. A patient with suspicion on epidemic processes do those formations relate? typhus was admitted to the hospital. Some A. Metaplasia arachnids and insects have been found in B. Inflammation his flat. Which of them may be a carrier of C. Regeneration the pathogen of epidemic typhus? D. Dystrophy A. Bed-bugs E. Hyperemia B. Houseflies 10. A patient with injured muscles of C. Lice the lower extremities was admitted to D. Spiders the traumatological department. Due to E. Cockroaches what cells is reparative regeneration of the 5. A 27- year-old woman has used penicillin muscle fibers and restoration of the muscle containing eye drops. In a few minutes itching, function possible? skin burning. lips and eyelids edema. A. Satellite-cells whistling cough. decreasing BP appeared. B. Fibroblasts What antibodies can lead to this allergic C. Myoepithelial cells reaction? D. Myoblasts A. IgE and IgG E. Myofibroblasts B. IgM and IgD 11. A worker has decreased buffer capacity Co IgA and IgM of blood due to exhausting muscular work D. IgG and IgD The influx of what acid substance in the E. IgM and IgG blood can cause this symptom?
  • 2. A.3-phosphoglycerate the bars form three zones. Medulla consists B.l,3-bisphosphoglycerate of chromaffinocytes and venous sinusoids. C. Pyruvate Which organ has these morphological D. α-ketoglutarate features? E. Lactate A. Thyroid 12. A 63-year-old woman developed B. Lymph node symptoms of rheumatoid arthritis. Their C. Kidney increase of which blood values indicators D. Adrenal gland could be the most significant in proving the E. Thymus diagnosis? 17. The study of the genealogy of a family A. Additive glycosaminoglycans with hypertrichosis (helix excessive pilosis) B. Acid phosphatase has demonstrated that this symptom C. Lipoproteids is manifested in all generations only in men D. General cholesterol and is inherited by son from his father. What E. R-glycosidase is the type of hypertrichosis inheritance? 13. A 50-year-old man has felt vague A. Y-linked chromosome abdominal discomfort within past 4 B. Autosome-dominant months. Physical examination revealed C. Autosome-recessive no lymphadenopathy, and no abdominal D. X-linked dominant chromosome masses or organomegaly at palpation. E. X-linked recessive chromosome Bowel sounds are heard. An abdominal CT 18. Succinate dehydrogenase catalyses the scan shows a 20 cm retroperitoneal soft tissue dehydrogenation of succinate. Malonic acid mass obscuring the left psoas muscle. HOOC-CH2-COOH is used to interrupt A stool specimen tested for occult blood is the action of this enzyme. Choose the inhibition negative. Which of the following neoplasms type: is this man most likely to have? A. Non-competitive A. Adenocarcinoma B. Limited proteolysis B. Lymphoma C. Allosteric C. Lipoma D. Competitive D. Melanoma E. Dephosphorylation E. Hamartoma 19. A patient has a malignisation of thoracic 14. The penetration of the irritable cell part of esophagus. What lymphatic nodes membrane for potassium ions has been are regional for this organ? increased during an experiment. What A. Nodi lymphatici mediastinales posteriores changes of membrane electric status can B. Nodi lymphatici paratrachealis occur? C. Nodi lymphatici prevertebralis A. Action potential D. Nodi lymphatici pericardiales laterales B. Depolarization E. Anulus lymphaticus cardiae C. Hyperpolarization 20. A patient with bronchial asthma had D. No changes been taking tablets which caused insomnia, E. Local response headache, increased blood pressure. What 15. Examination of a person revealed that medecine can cause such complications? minute volume of heart is 3500 mL, systolic A. Ephedrine volume is 50 mL. What is the frequency of B. Chromolin sodium cardiac contraction? C. Euphyline A. 50bpm D. Adrenaline B. 70bpm E. Izadrine C. 80bpm 21. Inhibition of alpha-motoneuron of the D. 60bpm extensor muscles was noticed after stimulation E. 90bpm of a-motoneuron of the flexor 16. A histological spacemen presents muscles during the experiment on the spinal parenchymal organ, which has cortex and column. What type of inhibition can be medulla. Cortex consists of epitheliocytes caused by this process? bars with blood capillaries between them; A. Depolarizational
  • 3. B. Reciprocal C. In the collecting duck C. Lateral D. In distal tubule of nephron D. Presynaptic E. In proximal tubule of nephron E. Recurrent 27. A patient suffering from trombophlebitis 22. During histological examination of the of deep veins suddenly died. The autopsy stomach it was found out that glands contained has shown freely lying red friable masses very small amount of pariental cells or with dim crimped surface in the trunk and they were totally absent. Mucose membrane bifurcation of the pulmonary artery. What of what part of the stomach was studied') pathologic process was revealed by the A. - morbid anatomist? B. Cardia A. Tissue embolism C. Body of stomach B. Fat embolism D. Pyloric part C. Tromboembolism E. Fundus of stomach D. Thrombosis 23. A businessman came to India from E. Embolism with foreign body South America. On examination the physician 28. On autopsy of a still-born infant found that the patient was suffering abnormalities have been revealed: ventricles from sleeping-sickness. What was the way are not separated, a single arterial trunk of invasion? originates from the right part. For what class A. After contact with a sick dogs of vertebrates is such heart construction B. Through dirty hands characteristic? C. As a result of mosquito's bites A. Fishes D. With contaminated fruits and vegetables B. Birds E. As a result of bug's bites C. Amphibian 24. A patient has been taking a mixture D. Reptiles prescribed by neuropathologist for E. Mammals neurasthenia for two weeks. The patient 29. A man after 1,5 litre blood loss has feels better but has developed coryza, suddenly reduced diuresis. TIle increased conjunctivitis, rash, inertia, decrease of secretion of what hormone caused such diuresis memory. She is diagnosed with bromizm. alteration? What should be prescribed to decrease the A. Parathormone symptoms? B. Corticotropin A. Glucose solution 5% C. Vasopressin B. Polyglucin D. Cortisol C. Asparcam E. Natriuretic D. Natrium chloride 30. Blood analysis of a patient showed signs E. - of HIV infection (human immunodeficiency 25. When the pH level of the stomach lumen virus). Which cells does HIV-virus primarily decreases to less than 3, the antrum of affect? the stomach releases peptide that acts in A. Specialized nervous cells (neurons) paracrine fashion to inhibit gastrin release. B. Cells that contain receptor IgM (B- This peptide is: lymphocytes) A. Gastrin-releasing peptide (GRP) C. Cells that contain receptor T4 (T-helpers) B. GIF D. Proliferating cells (stem hematoplastic C. Somatostatin cells) D. Vasoactive intestinal peptide (VIP) E. Mast cells E. Acetylcholine 31. Slime, blood and protozoa 30-200 microns 26. The low specific gravity of the secondary of length have been revealed in a urine (1002) was found out in the sick man's feces. The body is covered with cilias person. What is the most distant part of and has correct oval form with a little bit nephron where concentration of secondary narrowed forward and wide round shaped urine takes place? back end. On the forward end a mouth is visible. A. In the nephron's glomerulus In cytoplasm there are two nucleuses B. In ascending part of loop of Henle
  • 4. and two short vacuoles. For whom are the degrees due to gamma radiation. What type described attributes typical? of mutation took place in the DNA chain'? A. Balantidium A. Replication B. Trichomonas B. Inversion C. Intestinal amoeba C. Translocation D. Dysenteric amoeba D. Doubling E. Lamblia E. Deletion 32. A doctor administered Allopurinol to a 38. Periodic renal colics attacks are 26-year-old young man with the symptoms observed in a woman with primer! of gout. What pharmacological action of hyperparathyroidizm. Ultrasonic examination Allopurinol ensures therapeutical effect? revealed small stones in the kidneys. A. By increasing uric acid excretion What is the most plausible reason of the B. By inhibiting leucocyte migration into the stones's formation? joint A. Hyperphosphatemia C. By inhibiting uric acid synthesis B. Hyperuricemia D. By general analgetic effect C. Hypercalcemia E. By general anti-inflammatory effect D. Hyperkalemia 33. A person has steady HR not exceeding E. Hypercholesterinemia 40 bpm. What is the pacemaker of the heart 39. The gluconeogenesis is activated in the rhythm in this person? liver after intensive physical trainings .What A. Atrioventricular node substance is utilized in gluconeogenesis first B. His' bundle of all in this case: C. Sinoatrial node A. Glutamate D. Purkinye' libel'S B. Pyruvate E. Branches of His' bundle C. Lactate 34. In the ovary specimen colored with D. Glucose hematoxylin-eosin, follicle is determined E. Alanine where cubic-shaped follicle epithelium cells 40. From the nasopharynx of a 5-year- are placed in 1-2 layers, and scarlet covering old child 3 microorganism was excreted is seen around ovocyte. Name this follicle: which is identical to Corynebacterium diphtheriae A. Primordial dose according to morphological B. Secondary and biochemical signs. Microorganism does C. Mature . not produce exotoxin. As a result of what D. Atretic process can this microorganism become E. Primary toxigenic? 35. A denaturation of proteins can be found A. Phage conversion in some substances. Specify the substance B. Passing through the organism of the that is used for the incomplete denaturation sensative animals of hemoglobin: C. Chromosome mutation A. Toluene D. Growing with antitoxic serum B. Sulfuric acid E. Cultivation in the telluric environment C. Nitric acid 41. Necrosis focus was observed in the area D. Urea of hyperemia. and skin edema in a few hours E. Sodium hydroxide after burn. What mechanism strengthens 36. Ovarian tumour was diagnozed in a destructive effects in the inftamn1ation woman. Surgery should be perfomed. What area? ligament should be extracted by the surgeon A. Primary alteration to disconnect the ovary and the uterus? B. Emigration of lymphocytes A. Lateral umbilical ligament C. Diapedesis of erythrocytes B. Suspensory ligament of ovary D. Proliferation of fibroblasts C. The ovarial ligament E. Secondary alteration D. Round ligament of uterus 42. There is an inhibited coagulation in the E. Broad ligament of uterus patients with bile ducts obstruction, bleeding 37. Part of the DNA chain turned about 180 due to the low level of absorbtion of a
  • 5. vitamin. What vitamin is in deficiency? B. Escherichia A. Carotene C. Shigell B. E D. Streptococcus C. K E. Staphylococcus D. A 48. A 58-year-old female has undergone E. D surgery for necrotic bowel. Despite having 43. In case of enterobiasis acrihine - the been treated with antibiotics, on structural analogue of vitamin B2 - is postoperative day 5, she develops symptoms administered. (fever, hypotension, tachycardia, declining The synthesis disorder of which urine output, and confusion) consistent with enzymes does this medicine cause in septic shock. What hemodynamic support microorganisms? would be helpful? A. NAD-dependet dehydrogenases A. Fluid administration B. Cytochromeoxidases B. Dobutamine infusion C. Aminotransferases C. Fluids and Dobutamine infusion D. Peptidases D. Atropine administration E. FAD-dependent dehydrogenases E. Antibiotic administration 44. A patient with hypochromic anemia 49. A man died 8 days after the beginning has splitting and loss of hair, increased nail of the disease. He was diagnosed with brittling and taste alteration. What IS the dysentery. At the autopsy it was found mechanism of the symptoms development? out 'a thickened wall of the sigma and A. Decreased production of parathyrin rectum. fibrinous membrane on the surface B. Deficiency of vitamin A of mucous membrane. Histologically: there C. Deficiency of iron-containing enzymes is a deep necrosis of mucous membrane with D. Decreased production of thyroid infiltration of necrotic masses with fibrin. hormones What kind of colitis does correspond to the E. Deficiency of vitamin B12 changes? 45. Blood sampling for bulk analysis is A. Catarrhal recommended to be performed on an B. Diphtheritic empty stomack and in the morning. What C. Ulcerative changes in blood composition can occur if to D. Chronic perform blood sampling after food intake? E. Gangrenous A. Increased plasma proteins 50. The electronic microphoto of kidney B. Increased contents of leukocytes fragment has exposed afferent glomerular C. Reduced contents of erythrocytes arteriole, which has giant cells under its D. Increased contents of erythrocytes endothelium, containing secretory granules. E. Reduced contents of thrombocytes Name the type of these cells: 46. The high level of Lactate Dehydrogenase A. Interstitial (LDH) isozymes concentration showed the B. Mesangial increase of LDH-l and LDH-2 in a patient's C. Smoothmuscular blood plasma. Point out the most probable D. Juxtavascular diagnosis: E. Juxtaglomerular A. Myocardial infarction 51. When a patient with traumatic impairment B. Acute pancreatitis of the brain was examined, it was discovered C. Skeletal muscle dystrophy that he had stopped to distinguish D. Viral hepatitis displacement of an object on the skin. What E. Diabetes mellitus part of the brain was damaged? 47. Red colonies spread in the large quantity A. Posterior central gurus in the Endo culture medium were revealed B. Frontal zone on bacteriological stool examination of a 4month- C. Frontal central gurus old baby with the symptoms of acute D. Parietal zone of the cortex bowel infection. What microorganism can it E. Occipital zone of the cortex be? 52. A 30-year-old patient was hospitalized A. Salmonella due to bleeding of the facial artery. What
  • 6. place on the face has to be pressed to stop by what of the following cells is the most bleeding? typical for the process occuring here? A. The mandible's branch A. Monocytes B. The nose's back B. Limphocytes C. The molar bone C. Neutrophils D. The mental process D. Eosinophils E. The mandible's edge E. Basophils 53. A patient with complaints of 3-day-long 58. Marked increase of activity of MB-forms fever, general weakness, loss of appetite of CPK (creatinephosphokinase) and came to visit the infectionist. The doctor LDH-l was revealed by examination of the suspected enteric fever. Which method of patient's blood. What is the most probab1e laboratory diagnosis is the best to confirm pathology? the diagnosis? A. Rheumatism A. Detachment of myeloculture B. Hepatitis B. Detachment of urine culture C. Miocardial infarction C. Detachment of feces culture D. Pancreatitis D. Detachment of blood culture E. Cholecystitis E. Detachment of pure culture 59. A patient who came to the doctor 54. A 50-year-old patient with typhoid fever because of his infertility was administered was treated with Levomycetin, the next day to make tests for toxoplasmosis and chronic his condition became worse, temperature gonorrhoea. Which reaction should be rose to 39,6°. What caused the complication? performed to reveal latent toxoplasmosis A. Secondary infection addition and chronic gonurrhoea of the patient? B. Irresponsiveness of an agent to the A. IFA .. Immunofluorescence assay levomycetin B. RIHA - Reverse indirect hemagglutination C. The effect of endotoxin agent assay D. Allergic reaction C. (R)CFT- Reiter's complement fixation E. Reinfection test 55. A student is thoroughly summarising a D. RDHA - Reverse direct hemagglutination lecture. When his groupmates begin talking assay the quality of the summarising worsens E, Immunoblot analysis greatly. What type of inhibition in the 60. A 45-year-old woman suffers from cerebral cortex is the cause of it? allergic seasonal COr'/Z2 caused by the A. Differential ambrosia blossoming. What adipose cells B. Dying group stabilizer medicine can be used for C. Protective prevention of this disease? D. External A. Phencarol E. Delayed B. Diazoline 56. A 46 year-old man complains of difficult C. Ketotifen nose breathing. Mikulich cells, storage of D. Dimedrol epithelioid cells, plasmocytes, lymphocytes. E. Tavegyl hyaline balls are discovered in the biopsy 61. A patient after pathological process has material of the nose thickening. What is the a thickened alveolar membrane. The direct most likely diagnosis? consequence of the process will be the A. Virus rhinitis reduction of: B. Rhinovirus infection A. Reserve expiratiory capacity C. Meningococcal nasopharyngitis B. Minute respiratory capacity D. Allergic rhinitis C. Alveolar lung ventilation E. Scleroma Do Oxygen capacity of blood 57. A 16-year-old boy was performed an E. Diffuse lung capacity appendectomy. He has been hospitalized 62. A 59-year-old man has symptoms for right lower quadrant abdominal pain of parenchymatous jaundice and portal within 18 hours. The surgical specimen is hypertension. Histological examination edematous and erythematous. Infiltration of the puncture of the liver bioptate
  • 7. has revealed an affected beam-lobule blood pressure. Increasing of the osmotic structure, part of hepatocytes has signs of concentration and dispersity of protein fat dystrophy, port-portal connective tissue structures can be found in the intercellular septa with formation of pseudo-lobules, with fluid. What kind of edema are to be periportal lympho-macrophage infiltrations. observed in this case? What is the most probable diagnosis? A. Lymphogenic A. Liver cirrhosis B. Mixed B. Alcohol hepatitis C. Colloid-osmotic C. Toxic dystrophy D. Membranogenic D. Chronic hepatosis E. Hydrodynamic E. Viral hepatitis 68. A patient died from acute cardiac 63. The preventive radioprotector was given insufficiency. The histological examination to a worker of a nuclear power station. of his heart revealed the necrotized section What mechanism from the below mentioned in myocardium of the left ventricle, which is considered to be the main mechanism was separated from undamaged tissue of radioprotection? by the zone of hyperimic vessels, small A. Activation of oxidation reactions hemorrhages and leukocytic infiltration. B. Prevention of tissue's hypoxia What is the most likely diagnosis? C. Increasing of tissue blood supply A. Myocardial infarction D. Inhibition of free radicals formation B. Myocardial ischemic dystrophy E. Increasing of respiration C. Productive myocarditis 64. After the trauma. the patient's right D. Focal exudate myocarditis n.vagus was damaged. Which violation of E. Diffuse exudate myocarditis the cardiac activity is possible in this case? 69. A 56-year-old patient complaining of A. Violation of a conductivity in the right thirst and frequent urination was diagnosed auricle with diabete mellitus. Butamin was prescribed. B. Violation of the automatism of a How does the medicine act? atrioventricular A. It inhibits α-cells of Langergans' islets node B. It inhibits absorption of glucose in the C. Block of a conductivity in the atrioventricular intestines node C. It relieves transport of glucose through D. Violation of the automatism of a Kiss-Fleck the cells' membranes node D. It stimulates β-cells of Langergans' islets E. Arrhythmia E. It helps to absorb the glucose by the cells 65. A highly injured person has gradually died. of the organism tissues Please choose the indicator of biological 70. Galactosemia has been revealed in death: a child. Concentration of glucose in the A. Disarray of chemical processes blood has not considerably changed. What B. Autolysis and decay in the cells enzyme deficiency caused this illness? C. Absence of palpitation and breathing A. Galactokinase D. Absence of movements B. Galactose-1-phosphate uridyltransferase E. Loss of consciousness C. Hexokinase 66. During the experiment on the influence D. Amylo-1,6-glucosidase of chemical substances in the muscles the E. Phosphoglucomutase reaction of Ca2+ - pump is weakened. Which 71. During the endoscopy the inflammation phenomenum will be observed? of a major papilla of the duodenum A. Prolonged duration of the AP and the disturbances of bile secretion were B. Decreased AP found. In which part of duodenum were the C. Prolonged relaxation problems found? D. Decreased velocity of the AP distribution A. Lower horizontal part E. Activation of the sodium-potassium pump B. Ascendant part 67. Inflamation is characterised by increasing C. Descendent part penetration of vessels of microcirculation D. Upper horizontal part stream, increasing of their fluid dynamic E. Bulb
  • 8. 72. A 37-year-old man was admitted and small-grained. Microscopically: to the surgical department with the fibrinogenous symptoms of acute pancreatitis: vomiting, inflammation of serous and diarrhea, bradycardia, hypotention, mucous capsules, dystrophic changes of weakness, dehydration of the organism. parenchymatous organs, brain edema. What What medicine should be used first of all? complication can cause such changes of A. Ephedrine serous capsules and inner organs? B. Platyphylline A. Sepsis C. No-spa B. Uraemia D. Etaperazine C. Thrombopenia E. Contrycal D. DIC-syndrome 73. Hydroxylation of endogenous substrates E. Anemia and xenobiotics requires a donor of protons. 78. A 13-year-old girl with history of Which of the following vitamins can play asthma complained of cough, dyspnea this role? and wheezing. Her symptoms became so A. Vitamin B6 severe that her parents brought her to B. Vitamin A the emergency room. Physical examination C. Vitamin P revealed diaphoresis, dyspnea, tachycardia D. Vitamin C and tachypnea. Her respiratory rate was E. Vitamin E 42/min. pulse rate was 110 beats per minute, 74. Obturative jaundice developed in a and blood pressure was l30/70 mm 60-year-old patient because of malignant Hg. Choose from the following list the tumour of the big papillary of the duodenal. most appropriate drug to reverse the Lumen of what anatomical structure is bronchoconstriction rapidly: squeezed with tumour? A. Methylprednidsolone A. Right hepatic duct B. Salbutamol B. Common hepatic duct C. Beclomethasone C. Hepatopancreatic ampulla D. Cromolyn D. Cystic duct E. Ipratropium E. Left hepatic duct 79. Chronic glomerulonephritis was diagnosed 75. A couple came for medical genetic in a 34-year-old patient 3 years counseling. The man has hemophilia, the ago. Edema has developed within the last woman is healthy and there were no cases 6 monthes. What caused the edema? of hemophilia in her family. What is the risk A. Hyperosmolarity of plasma of having a sick child in this family? B. Hyperaldosteronism A.O C. Liver disfunction of protein formation B.25% D. Hyperproduction of vasopressin C.50% E. Proteinuria D.I00% 80. An isolated muscle of a frog is rhythmically E. 75% irritated with electric impulses. Every 76. Different functional groups can be next impulse is in a period of relaxation presented in the structure of L-amino acid's from the previus contraction. What radicals. Identify the group that is able contraction of the muscle occurs? to form ester bond: A. Continuous (smooth) tetanus A. -CH3 B. Asynchronous B.-CONH2 C. Single C.-OH D. Waved tetanus D. -NH2 E. Tonic E.-SH 81. The CNS stimulation produced by 77. For a long time a 49-year-old woman methylxanthines, such as caffeine, is most had suffered from glomerulonephritis which likely due to the antagonism of one of the caused death. The autopsy revealed following recertors: that the size of her kidneys was 7x3x2,5 A. Glycine receptors sm, weight 65,0 g, they were dense B. Adenosine receptors
  • 9. C. Glutamate receptors injured? D. GABA receptors A. The mind-brain E. Cholinergic muscarinic receptors B. The medulla oblongata 82. During complicated labour the C. The cerebellum symphysis pubis ruptured. What organ can D. The inter-brain be damaged mostly? E. The spinal cord A. Uterus 87. The alveolar ventilation of the patient is B. Rectum 5 L/min, the breath frequency is 10 per/min, C. Ovaria and the tidal volume is 700 ml. What is the D. Urinary blader patient's dead space ventilation? E. Uterine tubes A. 2,0 L/min 83. A patient with abscess of the cut B. 4,3 L/min wound applied to the traumatological c.- department. The wound was washed with D. 0,7 L/min 3% hydrogen peroxide to be cleaned from E. 1,0 L/min the pus. Foam was not observed. What 88. An old woman was hospitalized with caused inefficiency of the drug? acute pain, edema in the right hip joint; the A. Pus in the wound movements in the joint are limited. Which B. Shallow wound bone or part of it was broken? C. Inherited insufficiency of catalase A. Pubic bone D. Low concentration H202 B. The neck of the thigh E. Inherited insufficiency erythrocyte's C. Condyle of the thigh phosphatdehydrogenase D. Ischial bone 84. A woman suffering from dysfunctional E. The body of the thigh bone metrorrhagia was made a diagnostic abortion. 89. A patient, who suffers from congenital Histologically in the scrape there were erythropoietic porphyria, has skin a lot of small stamped glandulars covered photosensitivity. The accumulation of what with multirowed epithelium. The lumens of compound in the skin can cause it? some glandulars were cystically extended. A. Heme Choose the variant of general pathologic B. Uroporphyrinogen 2 process in the endometrium. C. Coproporphyrinogen 3 A. Glandular-cystic hyperplasia of D. Protoporphyrin endometrium E. Uroporphyrinogen 1 B. Neoplasm of endometrium 90. A patient died 3 days after the operation C. Atrophy of endometrium because of perforated colon with manifestations D. Metaplasia of endometrium of diffuse purulent peritonitis. E. Hypertrophic growth The autopsy revealed: colon mucos 85. An autopsy revealed: soft arachnoid membrane was thickened and covered with membrane of the upper parts of cerebral a fibrin film, isolated ulcers penetrated at hemisphere is plethoric, of yellowish-green different color, soaked with purulent and fibrose depth. The histology result: mucous exudate, it lookes like a cap. What disease is membrane necrosis, leukocytes infiltration characterised by these symtoms? with hemorrhages focuses. What disease A. Tuberculous meningitis complication caused the patient's death? B. Meningitis at typhus A. Dysentery C. Influenza meningitis B. Crohn's disease D. Meningitis at anthrax C. Amebiasis E. Meningococcal meningitis D. Typhoid 86. A 50 year-old patien was injured on E. Nonspecific ulcerative colitis the occipital region of the head. The closed 91. The formation of a secondary mediator skull's trauma was diagnosed. She was taken is obligatory in membrane-intracellular to the hospital. The medical examination: mechanism of hormone action. Point out deregulation of walking and balance. the substance that is unable to be a trembling of arms. What part of brain was secondary mediator:
  • 10. A. Glycerol B. Anconeus muscle B. Diacylglycerol C. Tendon of the triceps muscle C. CAMP D. Aponeurosis of biceps muscle D. CA2+ E. Brachioradial muscle ' E. Inositol-3,4,5-triphosphate 97. The reason of occurrence of some diseases 92. A 45-year-old man with domestic apper of an oral cavity is connected with arm injuiry came to the trauma unit. The structural peculiarities of its mucous objective data are: there are no extension, membrane. What morphological attributes adduction or pronation functions of characterize these features? the arm. What muscle damage caused this A. Transitional epithelium, no submucosa condition? B. No muscularis mucosa, stratified A. Subscapular squamous epithelium B. Teres major C. Transitional epithelium, no muscularis C. Subspinous mucosa D. Supraspinous D. Simple columnar ciliated epithelium E. Teres minor E. Well developed muscularis, no submucosa 93. The conjugated protein necessarily 98. A journalist's body temperature contains special component as a nonprotein has sharply increased in the morning part. Choose the substance that three weeks after his mission in India, can't carry out this function: it was accompanied with shivering and A. Thiamine pyrophosphate bad headache. A few hours later the B. Glucose temperature decreased. The attacks began C. HN03 to repeat in a day. He was diagnosed D. AMP with tropical malaria. What stage of E. ATP development of Plasmodium is infective for 94. M-r S presents all signs of the hepatic anopheles-female? coma: loss of consciousness, absence of A. Merozoites reflexes, cramps, convulsion, disorder of B. Gametocytes heart activity, recurrent (periodical) respiration. C. Sporozoites What are cerebrotoxical substances D. Microgamete which accumulate in blood under hepar E. Shizontes insufficiency? 99. A sick man with high temperature and A. Autoantibody a lot of tiny wounds on the body has been B. IL-l admitted to the hospital. Lice have been C. Necrosogenic substances found in the folds of his clothing. What disease D. Ammonia can be suspected in the patient? E. Ketonic body A. Plague 95. A 60-year-old patient was hospitalised B. Scabies to the surgical department because c. Malaria of infection caused by blue pus bacillus D. Epidemic typhus (Pseudomonas aeruginosa) which is sensative E. Tularemia to penicillin antibiotics. Indicate which of 100. The sterile Petri dishes and pipettes the given penicillins has marked activity to are necessary to prepare for microbiological the Pseudomonas aeruginosa? tests in bacteriological laboratory. What A. Oxacillin way of sterilization should be applied in this B. Benzylpenicillin case? C. Methicillin A. Dry-heat sterilization D. Phenoxymethylpenicillin B. Boiling E. Carbenicillin disodium C. Pasteurization 96. Usually the intravenous injection is done D. Tyndallization into median cubital vein because it is slightly E. Steam sterilizntion in autoclave movable due to fixation by the soft 101. An intraoperational biopsy of mammal tissues. What does it fix in the cubital fossa? gland has revealed the signs of atypical tissue A. Brachial muscle with disorder of parenchyma stroma
  • 11. proportion with domination of the last, E. Increased pressure in Bowman's capsule gland structures of different size and shape, 106. If strong oxidizers get into the lined with single-layer proliferative epithelium. bloodstream. a methemoglobin is formed. What is the most probable diagnosis? It is a compound, where iron (II) becomes A. Fibroadenoma iron (III). What has to be done to save the B. Papilloma patient? C. Noninfiltrative cancer A. He has to be calmed down and put to bed D. Mastitis B. Interchangeable hemotransfusion has to E. Infiltrative cancer be done 102. A 10-year-old child complains of C. He has to be given pure oxygen weakness, nausea, irritability. Helminthes D. Patient has to be exposed to the fresh air of white color and 5-10 mm long have been E. Respiratory centers have to be stimulated found on the underwear. On microscopy of 107. Patient 54 year-old, 5th day the scrape from the perianal folds achromic after surgical operation. Blood count: ova of the unsvmmetrical form have been Erythrocytes 3, 6* 1012/l, Hemoglobin 95 g/l, revealed. Which helminth is in the organism Erythrocyte's hemoglobin content (color of the child? index) 0,78; Leukocytes 16 * 109/l, Platelets A. Trichina 450*109/l. Blood picture: anizocytosis, B. Enterobins vermicularis poikilocytosis, C. Ascaris lumbricoides reticulocytes- 3,8%. What anemia D. Ancylostoma duodenalis does this patient have? E. Trichuris A. Acquired hemolytic anemia 103. A 68-year-old woman can not move B. Acute posthemorragic anemia her upper and lower right extremities after C. Hypoplastic anemia stroke. Muscle tone of these extremities and D. Anemia from iron deficiency reflexes are increased. There are pathological E. Chronic posthemorragic anemia reflexes. What form of the paralysis is it? 108. A woman who was sick with rubella A. Hemiplegia during the pregnancy gave birth to a deaf B. Dissociation child with hare lip and cleft palate. This C. Paraplegia congenital defect is an example of: D. Monoplegia A. Down's syndrome E. Tetraplegia B. Edward's syndrome 104. After breathing with poisonous steams C. Genocopy there is an increased quantity of slime in D. Phenocopy respiratory passages of a chemical production E. Patau's syndrome worker. What of respiratory tract epithelial 109. A 22-year-old patient was admitted cells participate in mucousa moistening? to the hospital with complaints of heavy A. Intercalated cells nasal breathing. During the examination B. Goblet cells of her nasal cavity the doctors found thickened C. Endocrine cells mucous membrane, a lot of mucus D. Langergans cells and nodular infiltrates without erosions in E. Fibroblasts the nose. The nasal rhinoscleroma was diagnosed. 105. A 16 year-old patient got numerous The biopsy was taken. What typical traumas in automobile accident. Now the morphological changes may be found? patient is having a shock. AP - 80/60 mm A. Granulomas with Virchow's cells Hg. daily urine volume 60-80 m!. What B. Granulomas with foreign body cells pathogenic mechanism leads to kidneys C. Interstitial inflammation function violation? D. Granulomas with Mikulicz's cells A. Decreased hydrostatic pressure m E. Granulomas with Langhan's cells glomerular capillaries 110. The alternate usage of dichlotiazide, B. Increased vasopressin blood concentration etacrin acid and lasex did not cause marked C. Trauma of the urinary bladder diuretic effect in the patient with marked D. Increased osmotic pressure in glomerular peripheral edema. The aldosterone level in capillaries the blood is increased. Indicate which medicine
  • 12. should be prescribed: E. Pathological anisocytosis A. Amilorid 116. Glomerular filtration rate (GFR) B. Urea increased by 20% due to prolonged starvation C. Clopamid of the person. The most evident cause of D. Mannit filtration changes under this conditions is: E. Spironolacton A. Increase of filtration coefficient 111. A patient with tissue trauma was taken B. Increase of penetration of the renal filter a blood sample for the determination of C. Decrease of oncotic pressure of blood blood clotting parameters. Specify the right plasma sequence of extrinsic pathway activation. D. Increase of systemic blood pressure A. III- VIIa - Xa E. Increase of renal plasma stream B. IV - VIII: TF - Xa 117. There is the change of teeth at the 6,8year- C. IV - VIIa-Xa old children: deciduous are replaced D. III- VIII: TF - Xa by permanent. What embrionic tissues are E. III-IV-Xa the sources of formation of permanent teeth 112. A damage of the atomic power plant tissues? reactor resulted in the run-out of radioelements. A. I, II brachial arches People in the superstandard radiation zone were B. Mesodermal epithelium and mesenhime radiated with approximately C. Ectodermal epithelium of a tooth plate 250-300 r. and were immediately and Inesenhin1c hospitalized. What changes in the blood D. Entodermal epithelium of a tooth plate count would be typical? and mesenhime A. Lymphopenia E. Entodermal epithelium and mesoderm B. Neutropenia 118. A 50 year-old patient had hemorrhage C. Leukopenia of the brain and was taken to the hospital. D. Thrombopenia The place of hemorrhage was revealed E. Anemia on the lateral hemispheres surfaces during 113. Decreased blood supply to the organs the medical examination. What artety was causes hypoxia that activates fibroblasts injured? function. Volume of what elements is A. The middle cerebral artery increased in this case? B. The posterior cerebral artery A. Nerve elements C. The anterior cerebral artery B. Lymphatic vessels D. The posterior communicating artery C. Vessels of microcircular stream E. The anterior conmmunicating artery D. Parenchymatous elements of the organ 119. While enrolling a child to school E. Intercellular substance Mantu's test was made to define whether 114. Live vaccine is injected into the human revaccination was needed. The test result is body. Increasing activity of what cells of negative. What does this test result mean? connective tissue can be expected? A. Absence of antitoxic immunity to the A. Fibroblasts and labrocytes tuberculosis B. Adipocytes and adventitious cells B. Presence of cell immunity to the C. Pigmentocytes and pericytes tuberculosis D. Plasmocytes and lymphocytes C. Presence of antibodies for tubercle bacillus E. Macrophages and fibroblasts D. Absence of antibodies for tubercle bacillus 115. In the blood of a 26-year-old man E. Absence of cell immunity to the 18% of erythrocytes of the spherical, ball-shaped, tuberculosis flat and thorn-like shape have been 120. Some diseases reveal symptoms revealed. Other eritrocytes were in the form of aldosteronism with hypertension and of the concavo-concave disks. How is this edema due to sodium retention in the phenomenon called? organism. What organ of the internal A. Erytrocytosis secretion is affected on aldosteronism? B. Physiological anisocytosis A. Adrenal glands C. Pathological poikilocytosis B. Pancreas D. Physiological poikilocytosis C. Hypophysis
  • 13. D. Testicle A. Macrophages, monocytes E. Ovaries B. NK-cells 121. Oval and round organelles with double C. Fibroblasts. T-Iymphocytes, B-lymphocytes wall are seen at the electron micrograph. D. T-Iymphocytes, B-Iymphocytes The outer membrane is smooth, the inner E. O-lymphocytes membrane folded into cristae contain 126. An autopsy has revealed that kidneys enzyme ATPase synthetase. These are: are enlarged, surface is large-granular A. Golgi complex because of multiple cavities with smooth B. Centrioles wall, which are filled with clear fluid. What Co Mitochondria kidney disease did the patient have? D. Lysosomes A. Pyelonephritis E. Ribosomes B. Infarction 122. A patient with encephalopathy was C. Glomerulonephritis admitted to neurological department. D. Polycystic kidney Correlation of increasing encephalopathy E. Necrotic nephrosis and substances absorbed by the 127. During the operation on the hip joint bloodstream from the intestines was revealed. of a 5-year-old child her ligament was What substances created in the damaged which caused bleeding. What ligament intestines can cause endotoxemia? was damaged? A. Biotin A. Iliofemoral B. Indole B. Ischiofemoral C. Ornithine C. The head of the thigh D. Butyrate D. Perpendicular of the acetabule E. Acetacetate E. Pubofemoral 123. A 19 year-old patient was diagnosed 128. A 57-year-old patient was admitted with appendicitis and was hospitalized. The to the gastroenterological department with surgical operation on ablating appendix suspicion of Zollinger-Ellison syndrom vermiformis is to be performed. What because of rapid increase of gastrin level in artery must be fixed to stop bleeding during the blood serum. What the most probable the surgical operation? disorder of the secretory function of the A. The colica media stomach here? B. The iliac A. Hypoacidity hyposecretion C. The colica dextra B. Hyperacidity hypersecretion D. The ileocolic artery C. Achylia E. The colica sinistra D. Hypoacidity hypersecretion 124. Methotrexate (structural analogue of E. Hyperacidity hyposecretion the folic acid which is competitive inhibitor 129. A 2-year-old child has got intestinal of the dihydrofolatreductase) is prescribed dysbacteriosis, which results in hemorrhagic for treatment of the malignant tumour. syndrome. What is the most likely cause of On which level does methotrexate hinder hemorrhage of the child? synthesis of the nucleic acids? A. Hypocalcemia A. Processing B. PP hypovitaminosis B. Mononucleotide synthesis C. Activation of tissue thromboplastin C. Transcription D. Fibrinogen deficiency D. Reparation E. Vitamin K insufficiency E. Replication 130. A patient suffering from thyrotoxicosis 125. A patient with clinical signs symptoms of vegetoasthenic syndrome of immunodeficiency has unchanged was revealed. What of the following would number and functional activity of show the histological appearance of a T and B lymphocytes. Dysfunction's thyroid gland being stimulated by defect of antigen-presentation to the thyroidstimulating immunocompetent cells was found during hormone (TSH)? investigation on the molecule level. Defect A. Columnar-shaped follicular cells of what cells is the most probable here? B. An abundance of colloid in the lumen of
  • 14. the follicle human blood sample is lower than normal. C. Increased numbers of parafollicular cells This leads to edema of tissues. What blood D. Decreased numbers of parafollicular function is damaged? capillaries A. Maintaining the Ph level E. Decreased numbers of follicular cells B. Maintaining the. blood sedimentation 131. The action of electric current on the system exitable cell caused depolarization of its C. All answers are correct membrane. Movement of what ions through D. Maintaining the oncotic blood pressure the membrane caused depolarisation? E. Maintaining the body temperature A. Ca2+ 136. In the microspecimen of red bone B. HCO3- marrow multiple capillares were revealed C. K+ through the walls of which mature blood D. CL- cells penetrated. What type of capillares is E. Na+ it? 132. A young man has an unpainfull A. Fenestrational formation without marked borders in the B. Somatical soft tissues of his thigh. On the tissue bioptate C. Visceral the formation lookes like flesh of D. Sinusoidal fish and consists of immature fibroblast-like E. Lymphatic cells with multiple mitosis growing through 137. A lung of a premature infant is the muscles. What is the most likely diagnosis? presented on electronic photomicrography A. Myosarcoma of biopsy material. Collapse of the alveolar B. Cancer wall caused by the deficiency of surfactant c. Myoma was revealed. Disfunction of what cells of D. Fibrosarcoma the alveolar wall caused it? E. Fibroma A. Secretory cells 133. A 46 year-old patient has complained B. Alveolar macrophages of headache, fatigue, thirst, pains in the spine C. Alveocytes type II and joints for the last 2 years. Clinically D. Fibroblasts observed disproportional enlargement of E. Alveocytes type I hands, feet, nose, superciliary arches. He 138. A patient had been taking glucocorticoids notes that he needed to buy bigger shoes for a long time. When the preparation three times. What is the main reason of such was withdrawn he developed the symptoms disproportional enlargement of different of disease aggravation, decreased blood parts of the body? pressure and weakness. What is the reason A. Increased sensitivity of the tissues to of this condition? insulin . A. Sensibilization B. Joints dystrophy development B. Appearance of adrenal insufficiency C. Cartilaginous tissue proliferation under C. Cumulation growth hormone influence n. Hyperproduction of ACTH D. Increased sensitivity of the tissues to E. Habituation growth hormone 139. A 2-vear-old child experienced E. Joints chronic inflammation development convulsions because of lowering calcium 134. A tissue sample of benign tumor was ions concentration in the blood plasma. studied under the electron microscope. A Function of what structure is decreased? lot of small (15-20 nm) spherical bodies, A. Parathyroid glands consisting of 2 unequal subunits were B. Hypophysis detected. These are: C. Adrenal cortex A. Microtubules D. Pineal gland B. Smooth endoplasmic reticulum E. Thymus C. Ribosomes 140. During the fetal period of the D. Mitochondria development in the vascular system of E. Golgi complex the fetus a large arterial (Botallo's) 135. The concentration of albumins in duct is functioning which converts into
  • 15. lig .arteriosum after birth. What anatomical B. Monocvtes formations does this duct connect? C. Eosinophiles A. Aorta and superior vena cava D. Lymphocytes B. Pulmonary trunk and aorta E. Platelets C. Right and left auricle 146. A 55-year-old patient with continuing D. Aorta and inferior vena cava ventricular arrhythmias was admitted to E. Pulmonary trunk and superior vena cava the hospital. The patient is taking timolol 141. The B cells of endocrine portion of drops for glaucoma, daily insulin injections pancreas are selectively damaged by alloxan for diabetes mellitus, and an ACE inhibitor poisoning. How will it be reflected in blood for hypertension. You have decided to use plasma? phenytoin instead of procainamide. What is A. The content of globulins decreases the reason? B. The level of sugar decreases A. The hypertensive effects of procainamide C. The content of albumins decreases would aggravate the hypertension D. The content of sugar increases B. The anticholinergic effect of procainamide E. The content of fibrinogen decrease would aggravate glaucoma 142. A consumptive patient has an open C. The cholinergic effects of procainamide pulmonary form of disease. Choose what would aggravate the diabetes sputum staining should be selected for finding D. The local anesthetic effect of procainamide out the tubercle (Koch's) bacillus? would potentiate diabetes A. Method of Burry-Gins E. The local anesthetic effect of procainamide B. Method of Ziel-Neelsen would aggravate the hypertension C. Method of Gram 147. Patient with diabetes mellitus experienced D. Method of Romanowsky-Giemsa loss of consciousness and convulsions E. Method of Neisser . after an injection of insulin. What might be 143. During the breakout of acute respiratory the result of biochemical blood analysis for infection in order to diagnose concentration of sugar? influenza the express-diagnosis, based on A. 3,3 mmol/L revealing of specific viral antigen in the B. 8,0 mmol/L examined material (nasopharyngial lavage), C. 10,0 mmol/L is carried out. Which reaction is used for D. 1.5 mmol/L this? E. 5,5 mmol/L A. Opsonization 148. Moving of the daughter chromatids to B. Complement binding the poles of the cell is observed in the mitotically C. Immunofluorescence dividing cell. On what stage of the D. Agglutination mitotic cycle is this cell? E. Precipitation A. Anaphase 144. A 62-year-old patient was admitted to B. Interfase the neurological department due to cerebral C. Prophase haemorrage. His condition is grave. There is D. Telophase evident progression of deep and frequent E. Metaphase breath that turnes into reduction to apnoea 149. A 25-year-old woman with red and and the cycle repeates. What respiration itchy eczematoid dermatitis visits your type has developed in the patient? office, She had a dental procedure one A. Gasping respiration day earlier with administration of a local B. Biot's respiration anesthetic. There were no other findings, C. Cheyne-Stockes respiration although she indicated that she had a history D. Apneustic respiration of allergic reactions. Which of the E. Kussmaul respiration following drugs is most likely involved? 145. Punctata hemorrhage was found out in A. Procaine the patient after application of a tourniquet. B. Etidocaine With disfunction of what blood cells is it C. Bupivacaine connected? D. Cocaine A. Neutrophiles E. Lidocaine
  • 16. 150. A mother of a newborn complains C. Autosomal dominant of her baby's constant belching with undigested D. X-linked recessive milk. Which developmental anomaly E. Y-linked is it an evidence of? 156. A patient's blood was analyzed and A. Esophageal atresia the decreased erythrocyte's sedimentation B. Esophageal fistula rate (ESR) was discovered. What disease C. Labium leporium from the listed below is accompanied with D. Faux lupinum decreased ESR? E. Anal atresia A. Vitamin B deficiency 151. Autopsy of the 58-year-old man hads B. Hepatitis revealed that mitral valve is deformed, thickened, C. Polycytemia does not totally close, Microscopically: D. Myocardial infarction centers of collagen fibers are eosinophilic, E. Splenomegaly have positive fibrin reaction. The 157. The pulmonalis embolism has suddenly most probable diagnosis is: developed in a 40 year-old patient with A. Mucoid swelling opened fracture of the hip. Choose the B. Fibrinoid inflammation possible kind of embolism. C. Amyloidosis A. Thrombus-embolus D. Fibrinoid swelling B. Air E. Hyalinosis C. Tissue 152. During surgical operation a blood D. Foreign body transfusion was made. The blood must be E. Fat checked to find antigens of some disease. 158. Where should the cathetor for What disease is expected to be found? evacuation of the lymph from the thoracic A. Virus of hepatitis A lymph duct be inserted? B. Enterovirus A. To the inferior vena cava C. Virus of hepatitis B B. To the right venous corner D. Virus of hepatitis E C. To the left inguinal vein E. Adenovirus D. To the left venous corner 153. Electrocardiogram of a 45-year-old E. To the superior vena cava man showed absence of P-wave in all the 159. A 42-year-old man who has been leads. What part of the conducting system is injured in a car accident is brought into the blocked? emergency room. His blood alcohol level on A. Atrioventricnlar node admission is 250 mg/dL. Hospital records B. Common branch of the bundle of His show a prior hospitalization for alcohol C. Purkinje's fibres related seizures. His wife confirms that he D. Branches of the bundle of His has been drinking heavily for 3 weeks. What E. Sinu-atrial node treatment should be provided to the patient 154. Nowadays about 50 minor bases have if he goes into withdrawal? been found ill the t-RNA structure besides A. None the main four nitrogenous bases. Choose B. Pentobarbital the minor nitrogenous base: C. Phenobarbital A. Adenine D. Diazepam B. Cytosine E. Phenytoin C. Cysteine 160. Synthesis of phospholipids is disordered D. Uracil under the liver fat infiltration. E. Dihydrouracil Indicate which of the following substances 155. A healthy woman has three sons can enhance the process of methylation affected by color blindness who were born during phospholipids synthesis? after her two marriages. Children both of A. Citrate her husbands are healthy. What is the most B. Ascorbic acid possible pattern of inheritance of this disease? C. Methionine A. Autosomal recessive D. Glycerin B. X-linked dominant E. Glucose
  • 17. 161. A patient with the symptoms of acute E. Mineralocorticoids alcoholic poisoning was brought to the 165. A 60-year-old patient was diagnosed hospital. What carbohydrates metabolism with hypothalamic lateral nuclei stroke. changes are typical for this condition? What changes in patient's behavior may be A. The g]uconeogenesis is increased in liver expected? B. The anaerobic glucose metabolism A. Depression predominates in muscles B. Aggressive behaviour C. The breakage of glycogen is increased in C. Thirst liver D. Unsatisfied hunger D. The anaerobic breakage of glucose is E. The rejection of food increased in muscles 166. A 55-year-old patient was hospitalized E. The gluconeogenesis velocity in liver is in result of the trauma of the medial decreased group of femoral muscles. What kind of 162. An individual is characterized by movements is the patient unable to do? rounded face, broad forehead, a mongollan A. Abduction of femur type of eyelid fold, flattened nasal bridge, B. Adduction of femur permanently open mouth. projecting C. Flexion of femur lower lip, protruding tongue, short neck, flat D. Extension of femur hands, and stubby fingers. What diagnosis E. Suppination of femur can be put to the patient? 167. The process of heart transplantation A. Klinefelter's syndrome determined the viability of myocardial cells. B. Supermales The determination of what myocardium C. Down's syndrome parameter is the most important'? D. Alkaptonuria A. Concentration of Ca-ions in heart vessels E. Turner's syndrome B. Concentration of oxygen in heart vessels 163. A 40-year-old woman has had a feeling of C. Rest potential of cardiomyocytes abdominal discomfort for the past D. Concentration of calcium-ions in myofibrils 8 months. On pelvic examination, there is E. Heart temperature the right adnexal mass. Abdominal CT scan 168. A woman who was infected with demonstrates a 7 Cm cystic mass involving toxoplasmosis during the pregnancy has a the right ovary with small areas of child with multiple congenital defects. This calcification. The uterus is normal in size. is a result of: The right fallopian tube and ovary have A. Recombination been removed surgically. Grossly, the mass B. Biological mutogenesis on sectioning is filled with abundant hair C. Teratogenesis and sebum. Microscopically, the mass has D. Chemical mutogenesis glandular spaces lined by columnar epithelium, E. Cancerogenesis squamous epithelium with hair follicles 169. The energy inputs of a healthy man cartilage, and dense connective tissue. What have been measured. In what position was type of tumour is it? the patient if his energy inputs were less A. Teratoma than the main exchange? B. Squamous cell carcinoma of ovary A. Nervous exertion C. Metastase of cervical carcinoma B. Easy work D. Melanoma C. Sleep E. Sarcoma of ovary D. Rest 164. A 19-year-old female suffers from E. Calmness tachycardia in rest condition, weight loss, 170. A patient after hypertension stroke excessive sweating, exophtalmos and irritability. does not have voluntary movements in his What hormone would you expect to right arm and leg with the increased muscle find elevated in her scrum? tone in these extremites. What type of disfunction A. Cortisol of nervous system is it? B. Insulin A. Peripheral paresis C. Thyroxine B. Peripheral paralysis D.ACTH C. Central paresis
  • 18. D. Central paralysis Malaria. What method of laboratory diagnostics E. Reflex paresis is necessary to use? 171. The patient has come to the hospital A. Immunological tests from the smelting workshop in the condition B. Blood examination of hyperthermia. What is the direct cause C. Urine examination of loss of consciousness at the heat stroke? D. Stool examination A. Decreased brain blood supply E. Examination of vaginal and urethral B. Dilatation of peripheral vessels discharge C. Decrease of heart output 176. The patient with pneumonia was D. Arterial pressure drop treated with antibiotics for a long period, E. Increased water loss through sweating After treatment patient complains of 172. Scraps of the mycelium of a fungus, frequent and watery stool, abdomenal pain. spores, air bubbles and fat drops were discovered What is the reason of intestine function disorder? on microscopy of the patient's A. Bacteria toxins influence hair excluded from the infected areas, What B. Intestinal disbacteriosis development fungus disease is characterised by this C. Antibiotics toxic influence on the GIT microscopic D. Autoimmune reaction development picture? E. Hereditary enzyme defect A. Epidermophytosis 177. Analeptical remedy of reflective type B. Favus from the H-cholinomimetics group was given C. Trichophytosis to the patient for restoration of breathing D. Microspory after poisoning with carbon monoxide, E. Sporotrichosis What medicine was prescribed to the patient? 173. A person was selling "homemade pork A. Adrenalin hydrochloride "sausages on the market. State sanitary B. Atropine sulphate inspector suspected falcification of the c. Mesaton sausages, What serological immune reaction D. Pentamin can identifiy food substance? E. Lobeline hydrochloride A. Complement- fixation test 178. A patient complains of frequent and B. Agglutination test difficult urination, Imperfection of what C. Precipitation test formation can cause it? D. Immunofluorescence test A. Prostate E. Indirect hemagglutination test B. Testicle adnexa 174. A 45-year-old man fell on the right C. Testicles knee and felt the acute pain in the joint On D. Sperm bubbles examination: severe edema on the anterior E. Bulb-uretic glands surface of the knee joint. Crunching sounds 179. A 10-year-old child complains of are heard, while moving the joint. Which weakness, nausea, irritability. Helminthes of bone is destroyed? while color and 5-10 mm long were found on A. Left epicondyle of the thigh the underwear. On microscopy of the scrape B. Knee-cap from the perianal folds achromic ovums of C. Head of the thigh bone unsymmetrical form were revealed. Indicate D. Neck of the thigh bone what helminth is parasiting on the child? E. Right epicondyle of the thigh A. Trichina 175. A patient has been brought to the B. Trichuris hospital with the complaints of headache, C. Enterobins vermicularis pain in left hypochondrium. He has been D. Ancylostoma duodenalis ill for 1,5 weeks, the sudden illness began E. Ascaris lumbricoides with the increase of body temperature up 180. A microscopic examination of the to 39,9°C. In 3 hours the temperature enlarged neck gland of a 14-year-old girl decreased and hydropoiesis began. The revealed destruction of the tissue structure attacks repeat rhythmically in 48 hours. of the node, absence of the lymph follicles, The patient had visited one an African sclerotic and necrosis parts. Cell Country. The doctors have suspected constitution of the node is polymorphous,
  • 19. lymphocites, eosinophiles, atypical cells C. Decreased lungs elasticity of the large size with mu1tiple-Iobule D. Inhibition of respiratory center nuclei (Beresovsky-Shternberg cells) and E. Excitation of respiratory center onenucleus large size cells are observed. 186. There is only One hormone among the What is the most likely diagnosis? neurohormones which refers to the derivatives A. Fungous mycosis of amino acids according to classification. B. Acute lympholeucosis Point it out: C. Berkitt's lymphoma A. Somatotropin D. Chronic lympholcucosis B. Oxytocin E. Lymphogranulomatous C. Vasopressin 181. Only one factor can influence the D. Melatonin charge of amino acid radicals in the active E. Thyroliberin centre of enzyme, Name this factor: 187. At the "laboratory experiment A. The surplus of a product the eukocyte culture was mixed with B. pH medium staphylococci. Neutrophile leukocytes C. Temperature engulfed and digested bacterial cells. This O. The presence of a competitive inhibitor processes are termed: E. Pressure A. Phagocytosis 182. The donor who had not donated the B. Facilitated diffusion blood for a long time was examined with C. Pinocytosis IFA method, Anti-HBs antibodies were D. Osmosis revealed, What does positive result of IFA E. Diffusion in this case mean? 188. A 50-year-old male farm worker has A. Chronic hepatitis C been brought to the emergency room. He B. Acute hepatitis C was found confused in the orchard and since C. Acute hepatitis B then has remained unconscious. His heart D. Previous hepatitis B rate is 45 and his blood pressure is 80/40 mm E. Chronic hepatitis B Hg. He is sweating and salivating profusely. 183. A patient has elbow joint trauma with Which of the following should be prescribed? avulsion of medial epicondyle of humerus. A. Pentamine What nerve can be damaged in this trauma? B. Norepinephrine A. Radial C. Physostigmine B. Medial cutaneous nerve of forearm D. Atropine C. Cardiac cutaneous nerve E. Proserine D. Musculocutaneous nerve 189. A young man felt sharp pain in the back E. Ulnar during active tightening on the horizontal 184. A 35-year-old man under the treatment bar. Objectively: pain while moving upper for pulmonary tuberculosis has acute-onset extremity, limited pronation and adduction of right big toe pain, swelling, and low-grade functions. Sprain of what muscle can be fever. The gouty arthritis was diagnosed and observed here? high serum uric acid level was found. Which A. M.subscapularis of the following antituberculosis drugs are B. M.latissimus dorsi known for causing high uric acid levels? C. M.romboideus major A. Rifampicin D. M.levator scapulae B. Aminosalicylic acid E. M.trapezius C. Thiacetazone 190. Intrapleural pressure is being measured D. Pyrazinamide in a person. In what phase does a person E. Cycloserine hold his breath if the pressure is - 25 cm 185. X-rav examination discovered lungs H20? emphysema in the patient. What is the A. Forced expiration reason of short breath development in this B. Quiet inspiration case? c.- A. Increased lungs elasticity D. Forced inspiration B. Decreasing of alveoli receptors sensitivity E. Quiet expiration
  • 20. 191. To anaesthetize the manipulation between tibia and fibula . What kind of bone related to burn surface treatment, a patient connection was injured? was intravenously injected a medication A. Fontanel for short-acting narcosis. 1 minute B. Ligament later the patient being under anaesthesia C. Membrane had increased blood pressure, tachycardia, D. Suture increased tone of sceletal muscles; reflexes E. Gomphosis remained. After awakening the patient had 196. On autopsy a 35-year-old man the focus desorientation and visual hallucinations. of carnification 5 cm in diametre enclosed What medication was the patient injected? in a thin capsule was revealed in the second A. Nitrous oxide segment of the right lung. The focus consists B. Thiopental sodium of a tough dry friable tissue with a C. Ketamine dim surface. For what disease are these D. Diethyl ether morphological changes typical? E. Sombrevin A. Lung cancer 192. The patient with diabetes mellitus has B. Tumorous form of silicosis been delivered in hospital in the state of C. Postinftammatory pneumosclerosis unconsciousness. Arterial pressure is low. D. Tuberculoma The patient has acidosis. Point substances, E. Chondroma which accumulation in the blood results in 197. A I8-year-old patient came to the outpatient these manifestations: department wiith the complaints of A. Cholesterol esters bleeding trauma in the vestibule of his nose. B. High fatty acids On examination: the mechanical injure of C. Monosaccharides the mucous laver of the vestibule without D. Amino acids continuation into nasal cavity proper. What E. Ketone bodies is the boundary between the vestibule and 193. A patient with hypersecretion of the nasal cavity proper? gastric juices was recomended to exclude A. Nasal limen concentrated bouillons and vegetable B. Nostrils decoctions from the diet because of their C. Nasal septa stimulation of gastric secretion. What is D. Choanes dominating E. Nasal roller mechanism of stimulation of secretion 198. A 52 year-old patient with bronchial in this case? asthma was treated with glucocorticoids. A. Stimulation of excretion of secretion in the Fever reaction appeared as a result of duodenum postinjective B. Irritation of taste receptors abscess. The patient had subfebrile C. Stimulation of gastrin production by temperature, which didn't correspond G-cells to latitude and severity of inflamnmatory D. Irritation of mechanoreceptors of the process. Why did patient have low fever stomach reaction? E. Irritation of mechanoreceptors of the oral A. Inhibited endogen pyrogens production cavity B. Violation of heat-producing mechanisms 194. Pyruvate concentration in the patient's C. Thermoregulation center inhibition urine has increased 10 times from normal D. Inflammatory barrier formation in injection amount. What vitamin deficiency can be the place reason of this change: E. Violation of heat loss through lungs A. Vitamin Bl 199. Purulent endometritis developed in a woman B. Vitamin Bb after delivery. Treating with antibiotics C. Vitamin C inhibitors of murein synthesis was D. Vitamin A ineffective. Wide spectrum bactericidal E. Vitamin E antibiotic was administered to her. In 6 195. A 6-year-old child fell on the cutting hours temperature rapidly increased up object and traumatized soft tissues to 40°C with shiver. Muscle pains have
  • 21. appeared. BP dropped down to 70/40 mmHg. Oligura has developed. What is the main reason for the development of this condition? A. Toxic effect of preparation B. Endotoxic shock C. Internal bleedin D. Anaphylactic shock E. Bacteremia 200. A patient in three weeks after acute myocardial infarction has pain in the heart and joints and pneumonia. What is the main mechanism of development of postinfarction Dressler's syndrome? A. Autoimmune inflammation B. Resorption of enzymes from necrotized area of myocardium C. Secondary infection D. Ischemia of myocardium E. Thrombosis vessels